Compiled and circulated by Dr. Parimal Dua, Assistant Professor, Dept. of Physiology, Narajole Raj college Endocrine System: 1. Describe the anatomy of the Endocrine System. The endocrine system is a complex network of glands and organs. It uses to control and coordinate your body's metabolism, energy level, reproduction, growth and development, and response to injury, stress, and mood. 1) is located at the base of the brain. It makes hormones that control hormones released in the pituitary gland. The hypothalamus controls water balance, sleep, temperature, appetite, mood and reproductive behaviors, and blood pressure. 2) Pineal gland is located in the middle of the brain. It makes the melatonin. This hormone helps your body know when it's time to sleep. This hormone also regulates the timing of other functions throughout the body, such as when puberty starts. 3) Pituitary gland is located below the brain. It is often as small as a pea. But it controls many functions of the other endocrine glands. 4) and parathyroid are located in front of the neck, below the voice box (larynx). The thyroid plays a key role in the body's metabolism. The parathyroid helps regulate the body's calcium balance and bone strength. 5) Adrenal gland is located on top of each kidney. Like many glands, these work together with the hypothalamus and pituitary gland. The adrenal glands make and release corticosteroid hormones and (epinephrine). These maintain blood pressure and regulate metabolism. 6) Pancreas is located across the back of the belly (abdomen), behind the stomach. It plays a role in digestion and hormone production. Hormones made by the pancreas include insulin and glucagon. These regulate blood sugar levels. 7) Ovaries are located on both sides of the uterus of woman, below the opening of the fallopian tubes. The ovaries contain the egg cells needed for reproduction. They also make estrogen and progesterone. 8) Testes are located in a pouch that hangs suspended outside the male body. The testes make testosterone and sperm.

Page | 1 DSC1DT: Sensory Physiology, Endocrine and Reproductive System, Renal Physiology

Compiled and circulated by Dr. Parimal Dua, Assistant Professor, Dept. of Physiology, Narajole Raj college

Figure: Endocrine glands and cells are located throughout the body and play an important role in homeostasis.

2. Classify the hormone. Hormones are the chemical messengers which are produced from endocrine glands and reach their target organ through blood to perform a specific function (mainly stimulation). Hormones can be classified in four ways...... 1) According to the nature of hormones: a) Steroid Hormones: Derived from Cholesterol. The Hormones produced in Adrenal Cortex and The Sex Hormones. They have important functions in the body like water balance, sexual development, stress response etc. E.g.: Testosterone, Estrogen, Progesterone, Glucocorticosteriods, Mineralocorticosteroids etc. b) Amine Hormones: Derived from Amino acids.

Page | 2 DSC1DT: Sensory Physiology, Endocrine and Reproductive System, Renal Physiology

Compiled and circulated by Dr. Parimal Dua, Assistant Professor, Dept. of Physiology, Narajole Raj college i. Thyroid Hormones: Derived from the Amino acid Tyrosine, Contains

Iodine Element. E.g. Tetra-iodothyronine (T4), Tri-iodothyronine (T3), Parathormone, Calcitonin etc. ii. Catecholamines: Derived from Tyrosine, can also function as a neurotransmitter. E.g. Epineprhine (Adrenaline), Nor-epinephrine (Nor-adrenaline) etc. iii. Tryptophan-based Hormones: As the name suggests, derived from the Essential Amino acid Tryptophan. E.g. Melatonin, . iv. Glutamic acid-based Hormones: Histamine is an example. c) Peptide Hormones: Derived from peptides (made up of few amino acids). E.g. Oxytocin (8 amino acids), GnRH (10 amino acids) and vasopressin. d) Polypeptide Hormones: Derived from peptides (chains of < 100 amino acids in length). E.g. ACTH (39 amino acids) e) Protein Hormones: Derived from proteins (Polypeptide chains with > 100 amino acids). E.g. Insulin, glucagon, and Prolactin (198 amino acid) f) Glycoprotein Hormones: Derived from proteins with carbohydrate molecules. E.g. LH and FSH. g) Fatty acid Derivative Hormones or Eicosaniods: Most abundant precursor is the Essential Fatty Acid Arachidonic Acid. E.g. Prostaglandins (Kidney), Prostacyclins, Leukotrienes, Thromboxanes etc. 2) According to the solubility: a) Lipid-soluble Hormones: Soluble in Lipids, Can work on the cell directly by going through the bi-layer phospolipid membrane of cells. E.g. Steroid Hormones. b) Water-soluble Hormones: Soluble in Water. Has to work indirectly because of the lack of penetration through the cell membrane. E.g. Amino acid-based Hormones. 3) According to the target organ: a) Tropic Hormones: Tropic Hormones are hormones which stimulate other endocrine glands to work. Generally Released from Pituitary Glands.

Page | 3 DSC1DT: Sensory Physiology, Endocrine and Reproductive System, Renal Physiology

Compiled and circulated by Dr. Parimal Dua, Assistant Professor, Dept. of Physiology, Narajole Raj college E.g. Gonado-Tropic Hormones, Thyroid Stimulating Hormone, Somatotropic Hormone, Adreno-Cortico Tropic Hormones etc. b) Non-Tropic Hormones: These work on other cells except Endocrine Glands. E.g. Calcitonin, Adrenaline, Oxytocin, Insulin, Glucagon etc. 4) According to the nature of action: a) General Hormones: Growth hormones influence nearly all the body tissues, similar is the case with Thyroid and Insulin hormones. Hence, they fall in general category. b) Specific Hormones: These hormones affect functions of specific organs. E.g. FSH and Androgen. c) Local Hormones: Prostaglandins, Acetyl cholin and Histamine act locally to their site of production. 3. What are hormone receptors in cell signaling? A hormone receptor is a protein molecule that binds to a specific hormone. Hormone receptors are a wide family of proteins made up of receptors for thyroid and steroid hormones, retinoids and Vitamin D, and a variety of other receptors for various ligands, such as fatty acids and prostaglandins. Once bound, the hormone-receptor complex initiates multiple signaling pathways, which ultimately leads to changes in the behavior of the target cells. Hormones usually require receptor binding to mediate a cellular response. There are two main classes of hormone receptors...... 1) Receptors for peptide hormones tend to be cell surface receptors built into the plasma membrane of cells and are thus referred to as trans membrane receptors. An example of this is insulin. 2) Receptors for steroid hormones are usually found within the cytoplasm and are referred to as intracellular or nuclear receptors, such as testosterone. 4. Write down about hormonal signaling? The presence of hormone or multiple hormones enables a response in the receptor, which begins a cascade of signaling. The hormone receptor interacts with different molecules to induce a variety of changes, such as an increase or decrease of nutrient sources, growth, and other metabolic functions. These signaling pathways are complex mechanisms mediated by feedback loops where different signals activate and inhibit other signals. If a signaling pathway ends with the increase in production of a nutrient, that nutrient is then a signal back to the receptor that acts as a competitive inhibitor to prevent further

Page | 4 DSC1DT: Sensory Physiology, Endocrine and Reproductive System, Renal Physiology

Compiled and circulated by Dr. Parimal Dua, Assistant Professor, Dept. of Physiology, Narajole Raj college production. Signaling pathways regulate cells through activating or inactivating gene expression, transport of metabolites, and controlling enzymatic activity to manage growth and functions of metabolism. 1) Intracellular (nuclear receptors): Intracellular and nuclear receptors are a direct way for the cell to respond to internal changes and signals. Intracellular receptors are activated by hydrophobic ligands that pass through the cellular membrane. All nuclear receptors are very similar in structure, and are described with intrinsic transcriptional activity. Intrinsic transcriptional involves the three following domains: transcription-activating, DNA-binding, and ligand-binding. These domains and ligands are hydrophobic and are able to travel through the membrane. The movement of macromolecules and ligand molecules into the cell enables a complex transport system of intracellular signal transfers through different cellular environments until response is enabled. Nuclear receptors are a special class of intracellular receptor that specifically aid the needs of the cell to express certain genes. Nuclear receptors often bind directly to DNA by targeting specific DNA sequences in order to express or repress transcription of nearby genes. 2) Trans-membrane receptors: The extracellular environment is able to induce changes within the cell. Hormones, or other extracellular signals, are able to induce changes within the cell by binding to membrane-bound receptors. This interaction allows the hormone receptor to produce second messengers within the cell to aid response. Second messengers may also be sent to interact with intracellular receptors in order to enter the complex signal transport system that eventually changes cellular function. G-protein-coupled membrane receptors (GPCR) are a major class of transmembrane receptors. The features of G proteins include GDP/GTP binding, GTP hydrolysis and guanosine nucleotide exchange. When a ligand binds to a GPCR the receptor changes conformation, which makes the intracellular loops between the different membrane domains of the receptor interact with G proteins. This interaction causes the exchange of GDP for GTP, which triggers structural changes within the alpha subunit of the G protein. The changes interrupts the interaction of the alpha subunit with the beta–gamma complex and which results in a single alpha subunit with GTP bound and a beta–gamma dimer. The GTP–alpha monomer interacts with a

Page | 5 DSC1DT: Sensory Physiology, Endocrine and Reproductive System, Renal Physiology

Compiled and circulated by Dr. Parimal Dua, Assistant Professor, Dept. of Physiology, Narajole Raj college variety of cellular targets. The beta–gamma dimer also can stimulate enzymes within the cells for example, adenylate cyclase but it does not have as many targets as the GTP–alpha complex. Aiding gene expression: Hormone receptors can behave as transcription factors by interacting directly with DNA or by cross-talking with signaling pathways. This process is mediated through co-regulators. In the absence of ligand, receptor molecules bind corepressors to repress gene expression, compacting chromatin through histone deacetylatase. When a ligand is present, nuclear receptors undergo a conformational change to recruit various coactivators. These molecules work to remodel chromatin. Hormone receptors have highly specific motifs that can interact with coregulator complexes. This is the mechanism through which receptors can induce regulation of gene expression depending on both the extracellular environment and the immediate cellular composition. Steroid hormones and their regulation by receptors are the most potent molecule interactions in aiding gene expression. Problems with nuclear receptor binding as a result of shortages of ligand or receptors can have drastic effects on the cell. The dependency on the ligand is the most important part in being able to regulate gene expression, so the absence of ligand is drastic to this process. For example, estrogen deficiency is a cause of osteoporosis and the inability to undergo a proper signaling cascade prevents bone growth and strengthening. Deficiencies in nuclear receptor-mediated pathways play a key role in the development of disease, like osteoporosis. 5. Describe the mechanism of action of hormone ? A hormone is a secreted chemical messenger that enables communication between cells and tissues throughout the body. Hormones are secreted by the glands of the endocrine system and they serve to maintain homeostasis and to regulate numerous other systems and processes, including reproduction and development. Signaling of hormone: The glands of the endocrine system secrete hormones directly into the extracellular environment. The hormones then diffuse to the bloodstream via capillaries and are transported to the target cells through the circulatory system. This allows hormones to affect tissues and organs far from the site of production or to apply systemic effects to the whole body.

Page | 6 DSC1DT: Sensory Physiology, Endocrine and Reproductive System, Renal Physiology

Compiled and circulated by Dr. Parimal Dua, Assistant Professor, Dept. of Physiology, Narajole Raj college Hormone-producing cells are typically specialized and reside within a particular endocrine gland, such as thryocytes in the thyroid gland. Hormones exit their cell of origin through the process of exocytosis or by other means of membrane transport. Cellular recipients of a particular hormonal signal may be one of several cell types that reside within a number of different tissues. This is so in the case of insulin, which triggers a diverse range of systemic physiological effects. Different tissue types may also respond differently to the same hormonal signal. As a result, hormonal signaling is elaborate and hard to dissect. Hormones activate target cells by diffusing through the plasma membrane of the target cells (lipid-soluble hormones) to bind a receptor protein within the cytoplasm of the cell, or by binding a specific receptor protein in the cell membrane of the target cell (water-soluble proteins). In both cases, the hormone complex will activate a chain of molecular events within the cell that will result in the activation of gene expression in the nucleus. The reaction of the target cells may then be recognized by the original hormone- producing cells, leading to a down-regulation in hormone production. This is an example of a homeostatic negative feedback loop.

Figure: Nuclear hormone receptors are activated by a lipid-soluble hormone such as estrogen, binding to them inside the cell. Lipid-soluble hormones can cross the plasma membrane.

Page | 7 DSC1DT: Sensory Physiology, Endocrine and Reproductive System, Renal Physiology

Compiled and circulated by Dr. Parimal Dua, Assistant Professor, Dept. of Physiology, Narajole Raj college Steps of Hormonal Signaling:  Biosynthesis of a particular hormone in a particular tissue.  Storage and secretion of the hormone.  Transport of the hormone to the target cells, tissues, or organs.  Recognition of the hormone by an associated cell membrane or an intracellular receptor protein.  Relay and amplification of the received hormonal signal via a signal transduction process.  Potential feedback to a hormone-producing cell.

Figure: Water-soluble hormones, such as epinephrine, bind to a cell-surface localized receptor, initiating a signaling cascade using intracellular second messengers.

6. Describe the basic concept of regulation of hormone actions. Hormone production and release are primarily controlled by negative feedback. In negative feedback systems, a stimulus elicits the release of a substance; once the substance reaches a certain level, it sends a signal that stops further release of the substance. In this way, the concentration of hormones in blood is maintained within a narrow range. For example, the amount of glucose in the blood controls the secretion of insulin and glucagons via negative feedback. During hormone regulation, hormones are released, either directly by an endocrine gland or indirectly through the action of the hypothalamus of the brain, which stimulates other endocrine glands to release hormones in order to maintain homeostasis. The hormones activate target cells, which initiate

Page | 8 DSC1DT: Sensory Physiology, Endocrine and Reproductive System, Renal Physiology

Compiled and circulated by Dr. Parimal Dua, Assistant Professor, Dept. of Physiology, Narajole Raj college physiological changes that adjust the body conditions. When normal conditions have been recovered, the corrective action – the production of hormones – is discontinued. Thus, in negative feedback, when the original (abnormal) condition has been repaired, or negated, corrective actions decrease or discontinue. In another example of hormone regulation, the signals the thyroid to release thyroid hormones. Increasing levels of these hormones in the blood then give feedback to the hypothalamus and anterior pituitary to inhibit further signaling to the thyroid gland, as illustrated in Figure 1.

Figure 1: The anterior pituitary stimulates the thyroid gland to release thyroid hormones T3 and T4. Increasing levels of these hormones in the blood results in feedback to the hypothalamus and anterior pituitary to inhibit further signaling to the thyroid gland.

Page | 9 DSC1DT: Sensory Physiology, Endocrine and Reproductive System, Renal Physiology

Compiled and circulated by Dr. Parimal Dua, Assistant Professor, Dept. of Physiology, Narajole Raj college There are three mechanisms by which endocrine glands are stimulated to synthesize and release hormones: humoral stimuli, hormonal stimuli, and neural stimuli. 1. Humoral Stimuli: The term “humoral” is derived from the term “humor,” which refers to bodily fluids such as blood. A humoral stimuli refers to the control of hormone release in response to changes in extracellular fluids such as blood or the ion concentration in the blood. For example, a rise in blood glucose levels triggers the pancreatic release of insulin. Insulin causes blood glucose levels to drop, which signals the pancreas to stop producing insulin in a negative feedback loop. 2. Hormonal Stimuli: Hormonal stimuli refers to the release of a hormone in response to another hormone. A number of endocrine glands release hormones when stimulated by hormones released by other endocrine glands. For example, the hypothalamus produces hormones that stimulate the anterior portion of the pituitary gland. The anterior pituitary in turn releases hormones that regulate hormone production by other endocrine glands. The anterior pituitary releases the thyroid-stimulating hormone, which then stimulates the thyroid

gland to produce the hormones T3 and T4. As blood concentrations of T3 and

T4 rise, they inhibit both the pituitary and the hypothalamus in a negative feedback loop. 3. Neural Stimuli: In some cases, the nervous system directly stimulates endocrine glands to release hormones, which is referred to as neural stimuli. Recall that in a short-term stress response, the hormones epinephrine and are important for providing the bursts of energy required for the body to respond. Here, neuronal signaling from the sympathetic nervous system directly stimulates the adrenal medulla to release the hormones epinephrine and norepinephrine in response to stress. 7. How does feedback mechanism regulate the hormone secretion? The feedback mechanism of hormones is the mechanism through which the balance of hormone in the blood/body is maintained. The increase or decrease in the concentration of that hormone can either stimulate the secretion of that particular hormone or inhibit the hormone secretion. This is called feedback.

Page | 10 DSC1DT: Sensory Physiology, Endocrine and Reproductive System, Renal Physiology

Compiled and circulated by Dr. Parimal Dua, Assistant Professor, Dept. of Physiology, Narajole Raj college There can be two type of feedback. One is called Positive feedback, and other is called negative feedback. The positive feedback stimulates the secretion or production of the hormone. On the counterpart, the negative feedback inhibits the secretion of the hormone. E.g. When we eat carbohydrate rich food, the glucose level in the blood is increase. As the Blood Glucose level rise, the pancreas will secrete Insulin. This Insulin will signal the cells to take up the blood glucose. Hence, the glucose level decreases in the blood. Now, if the insulin is still present in the blood, then more and more glucose will be transported inside the cell, and there would be a scarcity of the glucose in the blood. Hence to prevent this, negative feedback is generated due to low glucose level which would inhibit the insulin secretion in the blood. Positive feedback: Positive feedback occurs when a product feeds back to increase its own production. This causes conditions to become increasingly extreme. In this system, the output enhances the original stimulus. A good example of a positive feedback system is child birth. During labor, a hormone called oxytocin is released that intensifies and speeds up contractions. An example of positive feedback is milk production by a mother for her baby. As the baby suckles, nerve messages from the nipple cause the pituitary gland to secrete prolactin. Prolactin, in turn, stimulates the mammary glands to produce milk, so the baby suckles more. This causes more prolactin to be secreted and more milk to be produced. Negative feedback: Negative feedback occurs when a product feeds back to decrease its own production. This type of feedback brings things back to normal whenever they start to become too extreme. The thyroid gland is a good example of this type of regulation. It is controlled by the negative feedback loop. The hypothalamus secretes thyrotropin-releasing hormone, or TRH. TRH stimulates the pituitary gland to produce thyroid- stimulating hormone, or TSH. TSH, in turn, stimulates the thyroid gland to secrete its hormones. When the level of thyroid hormones in blood increases above a certain threshold, the hormones will feedback to stop the hypothalamus from secreting TRH and the pituitary from secreting TSH. Inhibition of TRH secretion leads to shut-off of TSH secretion, which leads to shut-off of thyroid hormone secretion. Without the stimulation of TSH, the thyroid gland stops

Page | 11 DSC1DT: Sensory Physiology, Endocrine and Reproductive System, Renal Physiology

Compiled and circulated by Dr. Parimal Dua, Assistant Professor, Dept. of Physiology, Narajole Raj college secreting its hormones. Soon, the level of thyroid hormone starts to fall too low. As thyroid hormone levels decay below the threshold, negative feedback is relieved, TRH secretion starts again, leading to TSH secretion.

Figure: The thyroid gland is regulated by a negative feedback loop. The loop includes the hypothalamus and pituitary gland in addition to the thyroid.

Another example is adrenal gland. Cortisol secretion by adrenal gland is regulated through the hypothalamus by releasing CRH (corticotropin releasing hormone) and pituitary by releasing ACTH (adrenocorticotropic hormone). Less tropic hormone (CRH and ACTH) secretion leads to less stimulation of cortisol secretion by cells of the zona fasciculata of the adrenal cortex. The usefulness of negative feedback inhibition is that it works to keep hormone levels within a particular appropriate physiological range. The reduced negative feedback inhibition means that more CRH and ACTH will be secreted. More ACTH will stimulate the remaining adrenal tissue to grow and to secrete more cortisol. This will have the effect of bringing cortisol back up towards its normal daily level of secretion. For an another example, an administration of estrogen and progesterone to the woman decreases the secretion of gonadotropins due to negative feedback inhibition.

Page | 12 DSC1DT: Sensory Physiology, Endocrine and Reproductive System, Renal Physiology

Compiled and circulated by Dr. Parimal Dua, Assistant Professor, Dept. of Physiology, Narajole Raj college Release of estrogen from the ovaries stimulates the secretion of GnRH from Hypothalamus which in turn stimulates the secretion of LH from anterior pituitary. When the levels of LH reaches above the normal levels in the blood it stimulates the release of progesterone from corpus luteum which causes the inhibition of the release of GnRH from the hypothalamus which in turn leads to inhibition of release of LH (product) from anterior pituitary and prevents the accumulation of LH (product)in blood.

Page | 13 DSC1DT: Sensory Physiology, Endocrine and Reproductive System, Renal Physiology

Compiled and circulated by Dr. Parimal Dua, Assistant Professor, Dept. of Physiology, Narajole Raj college Hypothalamo - Hypophysial axis: 8. What is neurohormone ? Give some examples. Neurohormones are chemical messenger molecules that are released by specialized neurons called neuroendocrine cells, but enter the bloodstream where they travel to distant target sites within the body. Therefore, neurohormones share characteristics with both neurotransmitters and hormones. Similar to neurotransmitters, neurohormones are released by neurons. Similar to hormones, neurohormones travel in the bloodstream. Neurohormones include neurohypophysial hormones, releasing hormones, adreno-medullary hormones, enteric hormones, enkephalins and other endorphins.  A first group of neurohormones, called neurohypophysial hormones are hormones produced by the neuroendocrine cells in the hypothalamus. The hypothalamus is the region of the brain located between the thalamus and the midbrain. It is made up of several small nuclei. Two well-known examples of neurohypophysial neurohormones are oxytocin and the antidiuretic hormone (vasopressin). These neurohormones are produced in the hypothalamic region of the brain and secreted into the blood by the neurohypophysis (part of the pituitary gland).  A second group of neurohormones, called releasing hormones (the first of which was chemically identified in 1969), also originates in the hypothalamus. The releasing hormones are released from the axon terminals that are transmitted within the neural cells to the median eminence (second locus in the brain), from which they pass in the bloodstream to the adenohypophysis, which also is a part of the anterior pituitary gland. There they either stimulate or inhibit the release of the various adenohypophysial hormones. Examples of releasing hormones are thyrotropin-releasing hormone, corticotropin-releasing hormone, growth hormone-releasing hormone, gonadotropin-releasing hormone, etc.  The neurohypophysial hormones are hormones produced by the neuroendocrine cells in the hypothalamus with axon terminals extending to the neurohypophysis. These hormones are stored inside the Herring bodies in the axon terminals and are secreted into the circulation to reach and

Page | 14 DSC1DT: Sensory Physiology, Endocrine and Reproductive System, Renal Physiology

Compiled and circulated by Dr. Parimal Dua, Assistant Professor, Dept. of Physiology, Narajole Raj college act on target cells. Examples of neurohypophysial hormones are oxytocin and vasopressin.  A third group of neurohormones called adreno-medullary neurohormones are catecholamines secreted from the adrenal medulla by the neuroendocrine cells, chromaffin cells. Examples of adreno-medullary hormones are epinephrine, norepinephrine, and .  A forth group of neurohormones called enteric neurohormones are neurohormones produced and released by enterochromaffin-like cells in the gastric glands of the stomach lining. These cells release histamine.  A fifth group of neurohormones includes the enkephalins and other endorphins, first observed in 1975 in the course of investigations of the mechanism of action of morphine and other analgesics. The endorphins are effective in relieving pain, a property apparently related to their function as neurotransmitters, passing nerve impulses from one neuron to another. Their neurohormonal activity is manifested by their stimulation of the secretion of somatotropin and vasopressin by an indirect process involving a site (other than the secretory neuron) in the central nervous system. 9. What is Hypothalamo-Hypophyseal System ? 1) Hypothalamo-hypophyseal portal system: Hypothalamo-hypophyseal portal system – From hypothalamus to anterior pituitary – Vascular link – Route for transport of hypothalamic releasing/release inhibiting hormones from hypothalamus to anterior pituitary The hypothalamic-hypophyseal portal circulation collects blood from capillaries originating in the hypothalamus and, through a plexus of veins surrounding the pituitary stalk, directs the blood into the anterior pituitary gland. This allows the neurohormones secreted by the neuroendocrine cells of the hypothalamus to be transported directly to the cells of the anterior pituitary. These hormones are largely, but not entirely, excluded from the general circulation.

Page | 15 DSC1DT: Sensory Physiology, Endocrine and Reproductive System, Renal Physiology

Compiled and circulated by Dr. Parimal Dua, Assistant Professor, Dept. of Physiology, Narajole Raj college

2) Hypothalamo-hypophyseal Tract: Hypothalamo-hypophyseal Tract – From hypothalamus to posterior pituitary (neurohypophysis) – Neural link – Route for transport of vasopressin (ADH) and oxytocin from hypothalamus to posterior pituitary The posterior lobe of pituitary is connected to the hypothalamus by a bridge of nerve axons called the hypothalamic–hypophyseal tract, along which the hypothalamus sends hormones produced by hypothalamic nerve cell bodies to the posterior pituitary for storage and release into the circulation.

Page | 16 DSC1DT: Sensory Physiology, Endocrine and Reproductive System, Renal Physiology

Compiled and circulated by Dr. Parimal Dua, Assistant Professor, Dept. of Physiology, Narajole Raj college 10. What is releasing factor ? Give some examples. Releasing hormones or releasing factor (the first of which was chemically identified in 1969) are peptide hormones, which are produced within the hypothalamus and transferred via the hypothalamo-hypophyseal portal veins to the adenohypophysis, where they regulate the synthesis or release of adenohypophyseal hormones. The main releasing hormones are as follows:  The thyrotropin-releasing hormone stimulates the release of thyrotropin or the thyroid-stimulating hormone (TSH) from the pituitary gland.  The corticotropin-releasing factor controls the release of the adrenocorticotrophic hormone (ACTH) from the pituitary gland.  The gonadotropin-releasing hormone controls the release of the two gonadotropin hormones, the luteinizing hormone (LH) and the follicle- stimulating hormone (FSH).  The growth hormone–releasing hormone (GHRH) controls the release of the somatotropin from the pituitary gland.  The prolactin-releasing factor controls the release of prolactin from the pituitary gland.  Examples of releasing hormones for exocrine hormones is gastrin-releasing peptide (GRP) which regulate gastrin production. 11. What is inhibiting factor ? Give some examples. Inhibiting hormones are hormones whose main purpose is to control the release of other hormones, by inhibiting their release. The main inhibiting hormones are as follows:  Somatostatin from the hypothalamus inhibits the pituitary gland’s secretion of growth hormone and thyroid stimulating hormone.  In addition, somatostatin is produced in the pancreas and inhibits the secretion of other pancreatic hormones such as insulin and glucagon. Somatostatin is also produced in the gastrointestinal tract where it acts locally to reduce gastric secretion, gastrointestinal motility and to inhibit the secretion of gastrointestinal hormones, including gastrin and secretin.  The hypothalamus uses dopamine or prolactin-inhibiting hormone (PIH) to inhibit the secretion of prolactin from the pituitary gland.  The hypothalamus uses follistatin to inhibit the secretion of follicle- stimulating hormone from the pituitary gland.  Myocytes use myostatin to tell each other to inhibit myogenesis.

Page | 17 DSC1DT: Sensory Physiology, Endocrine and Reproductive System, Renal Physiology

Compiled and circulated by Dr. Parimal Dua, Assistant Professor, Dept. of Physiology, Narajole Raj college  Melanocyte-inhibiting factor (melanostatin) inhibits release of other neuropeptides such as alpha-MSH and also has many other functions.  There is a neuropeptide called cortistatin and a class of steroidal cortistatins.  Gastric inhibitory polypeptide (GIP) regulates gastrin production. 12. What is tropic hormone ? Give some examples. Most tropic hormones are produced and secreted by the anterior pituitary. Tropic hormones from the anterior pituitary include Thyroid-stimulating hormone, Adrenocorticotropic hormone, Growth hormones.  Tropic hormones from the anterior pituitary include:  Thyroid-stimulating hormone (TSH or thyrotropin) – stimulates the thyroid gland to make and release thyroid hormone.[1]:718  Adrenocorticotropic hormone (ACTH or corticotropin) – stimulates the adrenal cortex to release glucocorticoids.[1]:718  Luteinizing hormone (LH) – stimulates the release of steroid hormones in gonads—the ovary and testes.[1]:718  Follicle-stimulating hormone (FSH) – stimulates the maturation of eggs and production of sperm.  The hypothalamus controls the release of hormones from the anterior pituitary by secreting a class of hypothalamic neurohormones called releasing and release-inhibiting hormones—which are released to the hypothalamo-hypophyseal portal system and act on the anterior pituitary. The hypothalamus secretes tropic hormones that target the anterior pituitary, and the thyroid gland secretes thyroxine, which targets the hypothalamus and therefore can be considered a tropic.

Page | 18 DSC1DT: Sensory Physiology, Endocrine and Reproductive System, Renal Physiology

Compiled and circulated by Dr. Parimal Dua, Assistant Professor, Dept. of Physiology, Narajole Raj college Pituitary gland: 13. Histological structure of anterior, middle and posterior lobes of pituitary gland. The pituitary gland undergoes rapid growth from birth to adult life to reach a weight of 500 mg. The adult gland has an anteroposterior diameter of 8 mm and a transverse diameter of 12 mm. There is a discrepancy between the size of the gland in males and females. During pregnancy, it almost doubles in size as the pars distalis enlarges. Pars distalis is a part of the anterior pituitary. It is bound superiorly by the diaphragma sellae, anteroinferiorly by the sphenoid sinus and laterally by the cavernous sinus. The optic chiasm lies anterosuperior to the gland. The tuber cinereum and median eminence of the hypothalamus give origin to an infundibulum. The tubular infundibulum connects the hypophysis to the brain. Due to the dual origin of the gland, they have a unique histological appearance. They are made up of anatomically and functionally distinct lobes called the anterior lobe (adenohypophysis), posterior lobe (neurohypophysis), and intermediate lobe. The pituitary gland is within the sella turcica or the hypophyseal fossa. This structure is present near the center at the base of the cranium and is fibro- osseous. The anatomical boundaries of the gland have clinical and surgical significance. Sella turcica is a concave indentation in the sphenoid bone. The reflections of the dura bound the fossa laterally and superiorly. Anatomy of Anterior Pituitary: The adenohypophyses constitute well-defined acini, consisting of cells that produce and secrete hormones. There are six cell lines, of which five are hormone-producing cell types called somatotropes, lactotropes, corticotropes, thyrotropes, and gonadotropes. Also, a nonhormone producing sixth cell type in the anterior pituitary called the folliculostellate cells. The anterior pituitary gland encompasses the following structures: 1) Pars Distalis: This is located at the distal part of the gland, and most of the hormones get secreted from this region. It forms the major bulk of the anterior pituitary. It is composed of follicles of varied sizes. Based on the staining methods used, the hormone-producing cells are classified below: a) Acidophils: They are composed of polypeptide hormones, and their cytoplasm stains red to orange in color. The somatotropes and lactotropes are the acidophils.

Page | 19 DSC1DT: Sensory Physiology, Endocrine and Reproductive System, Renal Physiology

Compiled and circulated by Dr. Parimal Dua, Assistant Professor, Dept. of Physiology, Narajole Raj college b) Basophils: They are composed of glycoprotein hormones and their cytoplasm stain blue to purple in color. The thyrotropes, gonadotropes, and corticotropes are the basophils. c) Chromophobes: They do not stain well. They may represent stem cells that are yet to differentiate into mature hormone-producing cells. 2) Pars Tuberalis: The tubular stalk is divided into pars tuberalis anteriorly and posteriorly. It extends from the pars distalis. The pars tuberalis encircles the infundibular stem, which is composed of unmyelinated axons from the hypothalamic nuclei. The hormones oxytocin and vasopressin accumulate in these axons, forming ovoid eosinophilic swellings along the infundibular stem. They make up the ‘herring bodies.’ 3) Pars Intermedia: This is present between the pars distalis and the posterior pituitary gland. It is made up of follicles containing a colloidal matrix and includes the remainder of the Rathke's pouch cleft. Though it is mostly nonfunctioning, they produce melanocyte-stimulating hormone, endorphins and have some pituitary stem cells. The hypothalamus is where the initial primary signal hormones get synthesized to stimulate the pituitary gland. Their synthesis is in the cell body of the neurons following which the axons project to terminate at the gland in the fenestrated portal capillaries. Then they travel via the bloodstream to the pituitary gland to stimulate the specific cells or inhibit it. Anatomy of Posterior Pituitary (neurohypophysis): Posterior Pituitary is a specialized neuroendocrine structure. The neurohypophysis is surrounded by the adenohypophysis and has a single nervous lobe, which is connected to the hypothalamus through the infundibulum. The neurohypophysis arises from the brain and also consists of three parts: 1) The infundibulum, a funnel-shaped downgrowth projection of the tuber cinereum; 2) The infundibular stem (pituitary stalk) and 3) The posterior lobe or pars nervosa (infundibular process) The neurohypophysis consists of unmyelinated axons of neurosecretory neurons which are supported by modified glial cells or pituicytes. Therefore, the neurohypophysis is not actually an endocrine gland since they contain axons that have originated from hypothalamic neurons, specifically the axon terminals of the magnocellular neurons of the paraventricular and supraoptic nuclei. Glial

Page | 20 DSC1DT: Sensory Physiology, Endocrine and Reproductive System, Renal Physiology

Compiled and circulated by Dr. Parimal Dua, Assistant Professor, Dept. of Physiology, Narajole Raj college cells called pituicytes encircle the axons. The pituicytes have elongated processes that run along with the axons; these are absent in a typical astrocyte and is due to the transcription factor expression TTF-1. The axons together form the hypothalamohypophyseal tract, which terminates near the posterior lobe sinusoids. The terminals of the axons are close to the blood vessels to aid in the secretion of the hormones (vasopressin and oxytocin). The precursor hormones are packed into secretory granules, called the herring bodies. These precursor hormones then get cleaved during transport to the posterior pituitary. Neurophysins are proteins that are essential for the posttranslational processing of the hormones. The posterior pituitary is not glandular, like the anterior pituitary. Thus they do not synthesize hormones.

Figure: Diagram of the anatomical components of the hypophysis.

14. Write the functions of pituitary gland. Functions of anterior pituitary: The following are the hormones produced and secreted from the anterior pituitary. 1) Adrenocorticotropic Hormone (ACTH): The release of this hormone from the gland is in response to the corticotropin-releasing hormone (CRH) from the hypothalamus. The CRH reaches the target location via the portal system and cleaves the proopiomelanocortin (POMC) into three major substances that are the ACTH, melanocyte-stimulating hormone, beta-endorphins. They then travel to reach the adrenal cortex, via the bloodstream to facilitate the

Page | 21 DSC1DT: Sensory Physiology, Endocrine and Reproductive System, Renal Physiology

Compiled and circulated by Dr. Parimal Dua, Assistant Professor, Dept. of Physiology, Narajole Raj college release of cortisol. The negative feedback from the cortisol regulates the CRH and ACTH. They aid in the secretion of glucocorticoids during stress. 2) Prolactin (PRL): This hormone is under the direct control of the hypothalamus. Dopamine inhibits the release of prolactin. The suckling of the baby in the postpartum period will inhibit the release of dopamine, thus disinhibiting prolactin release. When there is a drop in the dopamine levels due to disease or drugs, the patient will present with galactorrhea. Their primary function is to stimulate the growth of the mammary glands and participate in milk production. 3) Luteinizing Hormone (LH) and Follicle-Stimulating Hormone (FSH): The gonadotropin-releasing hormone (GnRH) that is secreted from the hypothalamus acts on the gonadotropin cells to secrete the LH and FSH. In males, the LH acts on the Leydig cells and secretes testosterone from the testes. The FSH acts on the Sertoli cells and secretes inhibin B for spermatogenesis. In females, the LH acts on the ovaries to initiate the production of the steroid hormone, and its surge causes ovulation. FSH acts on the granulosa cells and initiates follicular development for ovulation by the mature Graafian follicle. The steroid sex hormones regulate the LH and FSH through negative feedback. 4) Growth Hormone or Somatotropin (GH): The GH gets secreted from the somatotropes in response to the growth hormone-releasing hormone released from the hypothalamus. GH has anabolic properties and stimulates the growth of the cells in the body. The GH release is under the regulation of the negative feedback from the increased blood levels of GH and IGF-1. 5) Thyroid Stimulating Hormone (TSH): TSH secretion from the gland thyrotropes occurs in response to the thyrotropin-releasing hormone from the hypothalamus. This TSH acts on the thyroid gland to stimulate the release of T3 and T4. The TSH gets regulated by the blood levels of T3 and T4. Function of posterior pituitary The following are the two hormones released from the posterior pituitary. 1) Oxytocin: They participate in the milk let-down or milk ejection reflex during lactation, myoepithelial, and smooth muscle contraction, uterine contraction. This hormone is available for exogenous administration to patients with postpartum hemorrhage. Five IU of oxytocin is the recommended intravenous injection dosage to prevent postpartum

Page | 22 DSC1DT: Sensory Physiology, Endocrine and Reproductive System, Renal Physiology

Compiled and circulated by Dr. Parimal Dua, Assistant Professor, Dept. of Physiology, Narajole Raj college hemorrhage, and it is given following the delivery of the anterior shoulder of the fetus. 2) Arginine Vasopressin (AVP) or Antidiuretic Hormone (ADH): These hormones aid in the regulation of water content and prevents water depletion. It maintains the tonicity of the blood and blood pressure during an event of volume loss. The vascular smooth muscles express the V1 receptors, which, in response to the AVP, causes arteriolar contraction. The renal collecting duct and the tubular epithelium express V2 receptors, which in response to AVP, upregulate the aquaporin two channels and increases free water reuptake. 7. Write down the clinical significance of Pituitary gland. (Hypo and hyperactive states of pituitary gland) The following are some of the critical disease conditions associated with the pituitary gland. 1) Pituitary Adenoma: The most common pathology in the sellar region is the pituitary tumor. They classify into microadenomas (less than 10 mm) and macroadenomas (more than 10 mm). These macroadenomas can compress the adjacent structures. When it extends laterally, the cavernous sinus is compressed, producing ophthalmoplegia. The patients will present with diplopia due to cranial nerve compression. They may also present asymptomatically or with headaches. All the cell lines are capable of producing an adenoma. 2) Prolactinoma: This is the most common type of functioning secretory adenoma. They remain asymptomatic for an extended period until they cause compression or mass effect on the normal surrounding tissue causing hormonal dysfunction, visual changes, hydrocephalus, and hypogonadism. 3) Cushing Disease: This results from an ACTH secreting pituitary adenoma. They present with symptoms such as proximal myopathy, psychiatric disturbances, obesity, purple striae over the abdomen, extra fat around the neck termed buffalo hump, and hypertension. The standard method for resection of the tumor is the transsphenoidal adenectomy. The route is via the posterior wall of the sphenoid sinus, which forms the inferior border of the pituitary gland. 4) GH Secreting Adenoma: This adenoma produces a lot of GH, which can cause acromegaly or gigantism. The patient may present with carpal tunnel syndrome, proximal myopathy, and rarely hypertension or diabetes mellitus.

Page | 23 DSC1DT: Sensory Physiology, Endocrine and Reproductive System, Renal Physiology

Compiled and circulated by Dr. Parimal Dua, Assistant Professor, Dept. of Physiology, Narajole Raj college 5) Stalk Compression Syndrome: The patient presents with symptoms of hyperprolactinemia with raised prolactin concentration in the presence of a sellar or suprasellar mass compressing the stalk. 6) Pituitary Apoplexy: Pituitary adenomas that are often asymptomatic may enlarge in size and acquire additional arterial blood supply directly and undergo hemorrhagic infarction. The clinical presentation includes abrupt onset headache, visual symptoms like diplopia due to cavernous sinus compression, panhypopituitarism with low blood pressure, focal neurological deficits. The apoplexy triad includes headaches, visual changes, and vomiting. Clinicians can misdiagnose this condition as a subarachnoid hemorrhage. An emergency CT will show an enlarged pituitary fossa with some blood, and an MRI will confirm the diagnosis. 7) Sheehan Syndrome: This is due to the infarction and necrosis of the pituitary gland. It can be described as postpartum hypopituitarism, as postpartum hemorrhage is closely associated with its etiology. The gland enlarges during pregnancy, which causes the superior hypophyseal artery to get compressed. If the patient experiences a drop in her blood pressure during childbirth, this causes infarction and necrosis of the gland. The patient will commonly present with failure to lactate during the postpartum period, ultimately leading to a deficiency of the anterior pituitary hormones. 8) Lymphocytic Hypophysitis: This condition is commonly associated with pregnancy. It is an autoimmune condition with diffuse lymphocytic infiltration that destroys normal tissue with scaring. Imaging with an MRI shows homogenously enhancing and enlarged glands. 9) Granulomatous Hypophysitis: This is a non-caseating granuloma with Langerhans type giant cell. When it is present in the posterior lobe, it may be due to neurosarcoidosis. 10) Empty Sella Syndrome: There are two types of empty sella syndrome that are differentiated according to the cause. The primary empty sella syndrome is a defect in the diaphragma sellae that allows the contents above to herniate into the sella, thus compressing the gland. The secondary empty sella syndrome is due to causes such as tumors or surgery. This syndrome is associated with multiparous women, obesity, and benign intracranial hypertension. In multiparous women, this may be due to the repeated enlargement and involution of the gland leading to the gland becoming flattened. This condition must be

Page | 24 DSC1DT: Sensory Physiology, Endocrine and Reproductive System, Renal Physiology

Compiled and circulated by Dr. Parimal Dua, Assistant Professor, Dept. of Physiology, Narajole Raj college differentiated from craniopharyngioma and Rathke's cleft cyst as they may also present with a cyst within the same location. 11) Syndrome of Inappropriate Antidiuretic Hormone (SIADH): This is a condition with excessive production of antidiuretic hormone, which can result from various conditions such as nervous system disorders, neoplastic, ectopic sources such as paraneoplastic syndromes, head trauma, and drugs. The patients will present with altered mental status, seizures, or even more severe cases with coma. There will be hyponatremia due to dilution with free water. The urine analysis will show increased concentration and osmolality. 12) Craniopharyngiomas: Two variants are classified based on clinical and genetic features, which are the adamantinomatous (aCP) and papillary (pCP) craniopharyngiomas. The aCP occurs in children, and pCP is exclusive to adults. The embryonic theory proposed states that this develops from the ectopic embryonic remnants of Rathke's pouch. The metaplastic theory states that the squamous epithelium that is part of the gland undergoes metaplasia and transforms. 13) Rathke’s Cleft Cyst: This is a benign cyst that originates from the mucinous substances in the remnants of Rathke's pouch. The cyst contains the pink eosinophilic substance. Due to chronic inflammation, they may develop xanthogranulomas or cholesterol crystals. 14) Multiple Endocrine Neoplasia-1(MEN-1): This is a genetic, endocrine neoplastic syndrome. There is abnormal growth in the pituitary gland, parathyroid gland, and the pancreas. 8. What is Growth Hormone (Somatotropin) ? Growth Hormone ('GH) or somatotropin, also known as Human Growth Hormone ( HGH) in its human form, is a peptide hormone that stimulates growth, cell reproduction, and cell regeneration in humans and other animals. Growth hormone is composed with 190 amino acids and it is synthesized and secreted by cells called somatotrophs in the anterior pituitary. It is a major participant in control of several complex physiologic processes, including growth and metabolism. Growth hormone is also of considerable interest as a drug used in both humans and animals. 9. What are the physiological effects of growth hormone ? Growth hormone has two distinct types of effects:

Page | 25 DSC1DT: Sensory Physiology, Endocrine and Reproductive System, Renal Physiology

Compiled and circulated by Dr. Parimal Dua, Assistant Professor, Dept. of Physiology, Narajole Raj college 1) Direct effects are the result of growth hormone binding its receptor on target cells. Fat cells (adipocytes), for example, have growth hormone receptors, and growth hormone stimulates them to break down triglyceride and supresses their ability to take up and accumulate circulating lipids. 2) Indirect effects are mediated primarily by a insulin-like growth factor-I (IGF-I), a hormone that is secreted from the liver and other tissues in response to growth hormone. A majority of the growth promoting effects of growth hormone is actually due to IGF-I acting on its target cells. A. Effects on Growth Growth is a very complex process, and requires the coordinated action of several hormones. The major role of growth hormone in stimulating body growth is to stimulate the liver and other tissues to secrete IGF-I. IGF-I stimulates proliferation of chondrocytes (cartilage cells), resulting in bone growth. Growth hormone does seem to have a direct effect on bone growth in stimulating differentiation of chondrocytes. IGF-I also appears to be the key player in muscle growth. It stimulates both the differentiation and proliferation of myoblasts. It also stimulates amino acid uptake and protein synthesis in muscle and other tissues. B. Metabolic Effects Growth hormone has important effects on protein, lipid and carbohydrate metabolism. In some cases, a direct effect of growth hormone has been clearly demonstrated, in others, IGF-I is thought to be the critical mediator, and some cases it appears that both direct and indirect effects are at play. a) Protein metabolism: In general, growth hormone stimulates protein anabolism in many tissues. This effect reflects increased amino acid uptake, increased protein synthesis and decreased oxidation of proteins. b) Fat metabolism: Growth hormone enhances the utilization of fat by stimulating triglyceride breakdown and oxidation in adipocytes. c) Carbohydrate metabolism: Growth hormone is one of a battery of hormones that serves to maintain blood glucose within a normal range. Growth hormone is often said to have anti-insulin activity, because it supresses the abilities of insulin to stimulate uptake of glucose in peripheral

Page | 26 DSC1DT: Sensory Physiology, Endocrine and Reproductive System, Renal Physiology

Compiled and circulated by Dr. Parimal Dua, Assistant Professor, Dept. of Physiology, Narajole Raj college tissues and enhance glucose synthesis in the liver. Somewhat paradoxically, administration of growth hormone stimulates insulin secretion, leading to hyperinsulinemia. 9. How to control the secretion of growth hormone ? Production of growth hormone is modulated by many factors, including stress, exercise, nutrition, sleep and growth hormone itself. However, its primary controllers are two hypothalamic hormones and one hormone from the stomach: 1) Growth hormone-releasing hormone (GHRH) is a hypothalamic peptide that stimulates both the synthesis and secretion of growth hormone. 2) Somatostatin (SS) is a peptide produced by several tissues in the body, including the hypothalamus. Somatostatin inhibits growth hormone release in response to GHRH and to other stimulatory factors such as low blood glucose concentration. 3) Ghrelin is a peptide hormone secreted from the stomach. Ghrelin binds to receptors on somatotrophs and potently stimulates secretion of growth hormone. 4) Growth hormone secretion is also part of a negative feedback loop involving IGF-I. High blood levels of IGF-I lead to decreased secretion of growth hormone not only by directly suppressing the somatotroph, but by stimulating release of somatostatin from the hypothalamus. 5) Growth hormone also feeds back to inhibit GHRH secretion and probably has a direct (autocrine) inhibitory effect on secretion from the somatotroph. 6) Integration of all the factors that affect growth hormone synthesis and secretion lead to a pulsatile pattern of release. Basal concentrations of growth hormone in blood are very low. In children and young adults, the most intense period of growth hormone release is shortly after the onset of deep sleep. 10. What are the clinical significance of Growth hormone ? States of both growth hormone deficiency and excess provide very visible testaments to the role of this hormone in normal physiology. Such disorders can reflect lesions in either the hypothalamus, the pituitary or in target cells. A

Page | 27 DSC1DT: Sensory Physiology, Endocrine and Reproductive System, Renal Physiology

Compiled and circulated by Dr. Parimal Dua, Assistant Professor, Dept. of Physiology, Narajole Raj college deficiency state can result not only from a deficiency in production of the hormone, but in the target cell's response to the hormone. Clinically, deficiency in growth hormone or defects in its binding to receptor are seen as growth retardation or dwarfism. The manifestation of growth hormone deficiency depends upon the age of onset of the disorder and can result from either heritable or acquired disease. GH deficiency is one of the many causes of short stature and dwarfism. It results primarily from damage to the hypothalamus or to the pituitary gland during fetal development (congenital GH deficiency) or following birth (acquired GH deficiency). GH deficiency may also be caused by mutations in genes that regulate its synthesis and secretion. Affected genes include PIT-1 (pituitary-specific transcription factor- 1) and POUF-1 (prophet of PIT-1). Mutations in these genes may also cause decreased synthesis and secretion of other pituitary hormones. In some cases, GH deficiency is the result of GHRH deficiency, in which case GH secretion may be stimulated by infusion of GHRH. In other cases, the somatotrophs themselves are incapable of producing GH, or the hormone itself is structurally abnormal and has little growth-promoting activity. In addition, short stature and GH deficiency are often found in children diagnosed with psychosocial dwarfism, which results from severe emotional deprivation. When children with this disorder are removed from the stressing, nonnurturing environment, their endocrine function and growth rate normalize. GH deficiency is most often treated with injections of GH. The effect of excessive secretion of growth hormone is also very dependent on the age of onset and is seen as two distinctive disorders:  Giantism is the result of excessive growth hormone secretion that begins in young children or adolescents. It is a very rare disorder, usually resulting from a tumor of somatotropes. One of the most famous giants was a man named Robert Wadlow. He weighed 8.5 pounds at birth, but by 5 years of age was 105 pounds and 5 feet 4 inches tall. Robert reached an adult weight of 490 pounds and 8 feet 11 inches in height. He died at age 22.  Acromegaly results from excessive secretion of growth hormone in adults, usually the result of benign pituitary tumors. The onset of this disorder is typically insideous, occurring over several years. Clinical signs of acromegaly include overgrowth of extremities, soft-tissue swelling, abnormalities in jaw structure and cardiac disease. The excessive growth hormone and IGF-I also lead to a number of metabolic derangements, including hyperglycemia.

Page | 28 DSC1DT: Sensory Physiology, Endocrine and Reproductive System, Renal Physiology

Compiled and circulated by Dr. Parimal Dua, Assistant Professor, Dept. of Physiology, Narajole Raj college 11. Write the source of secretion and action of Oxytocin. Oxytocin is a polypeptide containing 9 aminoacids.  It has a half-life of about 6 minutes Source of Secretion: Oxytocin is secreted mainly by paraventricular nucleus and partly supraoptic nuclei of hypothalamus. Oxytocin is transported from hypothalamus to posterior pituitary through the nerve fibers of hypothalamo- hypophyseal tract. Oxytocin is stored in the nerve endings of hypothalamo- hypophyseal tract in the posterior pituitary. When suitable stimuli reach the posterior pituitary from hypothalamus, oxytocin is released into the blood. Oxytocin is secreted in both males and females. Action of Oxytocin: 1) Action on mammary glands ◦ Causes ejection of milk from the mammary glands. ◦ Oxytocin causes contraction of the myoepithelial cells and flow of milk from alveoli of mammary glands to the exterior through duct system and nipple. ◦ The process by which the milk is ejected from alveoli of mammary glands is called milk ejection reflex or milk letdown reflex. ◦ It is one of the neuro-endocrine reflexes. Milk ejection reflex:

Page | 29 DSC1DT: Sensory Physiology, Endocrine and Reproductive System, Renal Physiology

Compiled and circulated by Dr. Parimal Dua, Assistant Professor, Dept. of Physiology, Narajole Raj college As this reflex is initiated by the nervous factors and completed by the hormonal action, it is called a neuroendocrine reflex. During this reflex, large amount of oxytocin is released by positive feedback mechanism.

2) Action on uterus: a) Action on pregnant uterus: Throughout the period of pregnancy, oxytocin secretion is inhibited by estrogen and progesterone. At the end of pregnancy, the secretion of these two hormones decreases suddenly and the secretion of oxytocin increases. Oxytocin causes contraction of uterus and helps in the expulsion of fetus. It is also an example of neuro-endocrine reflex and positive feedback mechanism.

Page | 30 DSC1DT: Sensory Physiology, Endocrine and Reproductive System, Renal Physiology

Compiled and circulated by Dr. Parimal Dua, Assistant Professor, Dept. of Physiology, Narajole Raj college

b) Action on non-pregnant uterus: Action of oxytocin on non-pregnant uterus is to facilitate the transport of sperms through female genital tract up to the fallopian tube, by producing the uterine contraction during sexual intercourse.

Page | 31 DSC1DT: Sensory Physiology, Endocrine and Reproductive System, Renal Physiology

Compiled and circulated by Dr. Parimal Dua, Assistant Professor, Dept. of Physiology, Narajole Raj college 12. Write the source of secretion, action, regulation of secretion and applied physiology of Vasopressin. Vasopressin is also called as anti diuretic hormone. Vasopressin is a polypeptide containing 9 amino acids. Its half-life is 18 to 20 minutes. Source of Secretion: Vasopressin is secreted mainly by supraoptic and paraventricular nucleus in small quantity. From here, this hormone is transported to posterior pituitary through the nerve fibers of hypothalamo- hypophyseal tract, by means of axonic flow. Actions of Vasopressin: Antidiuretic hormone has two actions...... 1) Retention of water: Major function of ADH is retention of water by acting on kidneys. ◦ It increases the facultative reabsorption of water from distal convoluted tubule and collecting duct in the kidneys. Mode of action on renal tubules: i. Without ADH, the luminal membranes of the tubular epithelial cells of the collecting ducts are almost impermeable to water. ii. Immediately inside the cell membrane are a large number of special vesicles that have highly water permeable pores called aquaporins. iii. When ADH acts on the cell, it first combines with membrane receptors (V2 Receptors) that activate adenylyl cyclase and cause the formation of cAMP inside the tubular cell cytoplasm iv. This causes phosphorylation of elements in the special vesicles, which then causes the vesicles to insert into the apical cell membranes, thus providing many areas of high water permeability. v. All this occurs within 5 to 10 minutes. vi. Then, in the absence of ADH, the entire process reverses in another 5 to 10 minutes. vii. Thus, this process temporarily provides many new pores that allow free diffusion of water from the tubular fluid through the tubular epithelial cells and into the renal interstitial fluid 2) Vasopressor action: In large amount, ADH shows vasoconstrictor action. Particularly, causes constriction of the arteries in all parts of the body. Due to vasoconstriction, the blood pressure increases. ADH acts on blood vessels through V1A receptors. However, the amount of ADH required to cause the vasopressor effect is greater than the amount required to cause the antidiuretic effect. One of the stimuli for causing intense ADH secretion is decreased blood volume. This occurs especially strongly when the blood volume decreases 15 to 25 per cent or more; the secretory rate then sometimes rises to as high as 50 times normal. Mode of action: i. The atria have stretch receptors that are excited by overfilling. ii. When excited, they send signals to the brain to inhibit ADH secretion.

Page | 32 DSC1DT: Sensory Physiology, Endocrine and Reproductive System, Renal Physiology

Compiled and circulated by Dr. Parimal Dua, Assistant Professor, Dept. of Physiology, Narajole Raj college iii. Conversely, when the receptors are unexcited as a result of underfilling, the opposite occurs, with greatly increased ADH secretion. iv. Decreased stretch of the baroreceptors of the carotid, aortic, and pulmonary regions also stimulates ADH secretion.

Regulation of secretion: ADH secretion depends upon the volume of body fluid and the osmolarity of the body fluids. Potent stimulants for ADH secretion are: a) Decrease in the extracellular fluid (ECF) volume. b) Increase in osmolar concentration in the ECF.

Page | 33 DSC1DT: Sensory Physiology, Endocrine and Reproductive System, Renal Physiology

Compiled and circulated by Dr. Parimal Dua, Assistant Professor, Dept. of Physiology, Narajole Raj college

Applied physiology: Hypersecretion: Syndrome of Inappropriate Hypersecretion of Antidiuretic Hormone (SIADH) Hyposecretion: Diabetes insipidus SIADH: SIADH is the disease characterized by loss of sodium through urine due to hypersecretion of ADH  Causes: SIADH occurs due to cerebral tumors, lung tumors and lung cancers because the tumor cells and cancer cells secrete ADH. In normal conditions, ADH decreases the urine output by facultative reabsorption of water in distal convoluted tubule and the collecting duct. Urine that is formed is concentrated with sodium and other ions.  Signs and symptoms:  Loss of appetite  Weight loss  Nausea and vomiting  Headache  Muscle weakness, spasm and cramps

Page | 34 DSC1DT: Sensory Physiology, Endocrine and Reproductive System, Renal Physiology

Compiled and circulated by Dr. Parimal Dua, Assistant Professor, Dept. of Physiology, Narajole Raj college  Fatigue  Restlessness and irritability.  In severe conditions, the patients die because of convulsions and coma. Diabetes insipidus: Diabetes insipidus is a posterior pituitary disorder characterized by excess excretion of water through urine due to a defect in ADH secretion.  Causes: This disorder develops due to the deficiency of ADH, which occurs in the following conditions: . Lesion (injury) or degeneration of supraoptic and paraventricular nuclei of hypothalamus. . Lesion in hypothalamo-hypophyseal tract. . Atrophy of posterior pituitary. . Inability of renal tubules to give response to ADH hormone. (Nephrogenic diabetes insipidus)  Signs and symptoms:  Polyuria  Polydipsia  Dehydration

Page | 35 DSC1DT: Sensory Physiology, Endocrine and Reproductive System, Renal Physiology

Compiled and circulated by Dr. Parimal Dua, Assistant Professor, Dept. of Physiology, Narajole Raj college Thyroid gland: 13. Describe the location and anatomy of the thyroid gland A butterfly-shaped organ, the thyroid gland is located anterior to the trachea, just inferior to the larynx (Figure 1). The medial region, called the isthmus, is flanked by wing-shaped left and right lobes. Each of the thyroid lobes are embedded with parathyroid glands, primarily on their posterior surfaces. The tissue of the thyroid gland is composed mostly of thyroid follicles. The follicles are made up of a central cavity filled with a sticky fluid called colloid. Surrounded by a wall of epithelial follicle cells, the colloid is the center of thyroid hormone production, and that production is dependent on the hormones’ essential and unique component: iodine.

Figure: Thyroid Gland. The thyroid gland is located in the neck where it wraps around the trachea. (a) Anterior view of the thyroid gland. (b) Posterior view of the thyroid gland.

14. Describe the histological structure of thyroid gland. Thyroid consists of two lateral lobes and the connecting part called isthmus. It is a largest gland in body. Endodermal in origin. Gland secretes thyroid hormones called thyroxine. Structural unit of thyroid gland is follicle or acinus. Follicle consists of layer of simple epithelium, enclosing cavity called the follicular cavity. The cavity is usually filled with gel-like viscous iodine-rich material called colloid. Interfollicular spaces are filled by reticular connnective tissue, adipose tissue and blood vessels. Follicular cells: These are cuboidal epithelial cells with their basal ends resting on basement membrane. These cells show the changes in shape depending on state of gland. These cells exhibit squamous structure when thyroid gland is

Page | 36 DSC1DT: Sensory Physiology, Endocrine and Reproductive System, Renal Physiology

Compiled and circulated by Dr. Parimal Dua, Assistant Professor, Dept. of Physiology, Narajole Raj college inactive and columnar when the gland is hyperactive. The follicular cells show central or basal round nucleus with one or more excentric nuclei. The apical tips of cells extend microvilli in the cavity. Parafollicular cells: In Interfollicular spaces there are some special parafollicular cells. They found in singly or in groups. They secrete hormone thyrocalcitonin, which lowers the calcium level. Colloid: Cavity of thyroid follicle is filled with semi-fluid or gel like substance, called thyroid colloid. It is the endocrine secretion of epithelial cells and composed of nucleoproteins, thyroglubolin and proteolytic enzymes. Among the endocrine glands, thyroid is unique because it utilizes inorganic element iodine for the synthesis of its hormones.

15. Discuss the synthesis and release of triiodothyronine (T3) and thyroxine

(T4). Hormones are produced in the colloid when atoms of the mineral iodine attach to a glycoprotein, called thyroglobulin that is secreted into the colloid by the follicle cells. The following steps outline the hormones’ assembly: 1) Binding of TSH to its receptors in the follicle cells of the thyroid gland causes the cells to actively transport iodide ions (I–) across their cell membrane, from the bloodstream into the cytosol. As a result, the concentration of iodide ions “trapped” in the follicular cells is many times higher than the concentration in the bloodstream. 2) Iodide ions then move to the lumen of the follicle cells that border the colloid. There, the ions undergo oxidation (their negatively charged electrons – are removed). The oxidation of two iodide ions (2 I ) results in iodine (I2), which passes through the follicle cell membrane into the colloid.

Page | 37 DSC1DT: Sensory Physiology, Endocrine and Reproductive System, Renal Physiology

Compiled and circulated by Dr. Parimal Dua, Assistant Professor, Dept. of Physiology, Narajole Raj college 3) In the colloid, peroxidase enzymes link the iodine to the tyrosine amino acids in thyroglobulin to produce two intermediaries: a tyrosine attached to one iodine and a tyrosine attached to two iodines. When one of each of these intermediaries is linked by covalent bonds, the resulting compound

is triiodothyronine (T3), a thyroid hormone with three iodines. Much more commonly, two copies of the second intermediary bond, forming

tetraiodothyronine, also known as thyroxine (T4), a thyroid hormone with four iodines. These hormones remain in the colloid center of the thyroid follicles until TSH stimulates endocytosis of colloid back into the follicle cells. There, lysosomal

enzymes break apart the thyroglobulin colloid, releasing free T3 and T4, which diffuse across the follicle cell membrane and enter the bloodstream.

In the bloodstream, less than one percent of the circulating T3 and T4 remain

unbound. This free T3 and T4 can cross the lipid bilayer of cell membranes and

be taken up by cells. The remaining 99 percent of circulating T3 and T4 is bound to specialized transport proteins called thyroxine-binding globulins (TBGs), to albumin, or to other plasma proteins. This “packaging” prevents their free diffusion into body cells. When blood levels of T3 and T4 begin to decline,

bound T3 and T4 are released from these plasma proteins and readily cross the

membrane of target cells. T3 is more potent than T4, and many cells convert

T4 to T3 through the removal of an iodine atom. 15. Explain the regulation of the synthesis of thyroid hormones

The release of T3 and T4 from the thyroid gland is regulated by thyroid- stimulating hormone (TSH). Binding of TSH to its receptors on thyroid epithelial cells stimulates synthesis of the iodine transporter, thyroid peroxidase

and thyroglobulin. As shown in Figure 1, low blood levels of T3 and

T4 stimulate the release of thyrotropin-releasing hormone (TRH) from the hypothalamus, which triggers secretion of TSH from the anterior pituitary. In

turn, TSH stimulates the thyroid gland to secrete T3 and T4. The levels of TRH,

TSH, T3, and T4 are regulated by a negative feedback system in which

increasing levels of T3 and T4 decrease the production and secretion of TSH. The magnitude of the TSH signal also sets the rate of endocytosis of colloid - high concentrations of TSH lead to faster rates of endocytosis, and hence, thyroid hormone release into the circulation. Conversely, when TSH levels are low, rates of thyroid hormone synthesis and release diminish.

Page | 38 DSC1DT: Sensory Physiology, Endocrine and Reproductive System, Renal Physiology

Compiled and circulated by Dr. Parimal Dua, Assistant Professor, Dept. of Physiology, Narajole Raj college

Figure: Classic Negative Feedback Loop. A classic negative feedback loop controls the regulation of thyroid hormone levels.

16. Explain the role of thyroid hormones in the regulation of basal metabolism Functions of Thyroid Hormones:

The thyroid hormones, T3 and T4, are often referred to as metabolic hormones because their levels influence the body’s basal metabolic rate, the amount of energy used by the body at rest. When T3 and T4 bind to intracellular receptors located on the mitochondria, they cause an increase in nutrient breakdown and

the use of oxygen to produce ATP. In addition, T3 and T4 initiate the transcription of genes involved in glucose oxidation. Although these mechanisms prompt cells to produce more ATP, the process is inefficient, and an abnormally increased level of heat is released as a byproduct of these reactions. This so-called calorigenic effect (calor- = “heat”) raises body temperature.

Page | 39 DSC1DT: Sensory Physiology, Endocrine and Reproductive System, Renal Physiology

Compiled and circulated by Dr. Parimal Dua, Assistant Professor, Dept. of Physiology, Narajole Raj college Adequate levels of thyroid hormones are also required for protein synthesis and for fetal and childhood tissue development and growth. They are especially critical for normal development of the nervous system both in utero and in early childhood, and they continue to support neurological function in adults. As noted earlier, these thyroid hormones have a complex interrelationship with reproductive hormones, and deficiencies can influence libido, fertility, and other aspects of reproductive function. Finally, thyroid hormones increase the body’s sensitivity to catecholamines (epinephrine and norepinephrine) from the adrenal medulla by upregulation of receptors in the blood vessels. When levels of

T3 and T4 hormones are excessive, this effect accelerates the heart rate, strengthens the heartbeat, and increases blood pressure. Because thyroid hormones regulate metabolism, heat production, protein synthesis, and many other body functions, thyroid disorders can have severe and widespread consequences. 17. Discuss the disorders of Iodine Deficiency, Hypothyroidism, and Hyperthyroidism. Iodine Deficiency: As discussed above, dietary iodine is required for the

synthesis of T3 and T4. But for much of the world’s population, foods do not provide adequate levels of this mineral, because the amount varies according to the level in the soil, in which the food was grown, as well as the irrigation and fertilizers used. Marine fish and shrimp tend to have high levels because they concentrate iodine from seawater, but many people in landlocked regions lack access to seafood. Thus, the primary source of dietary iodine in many countries is iodized salt. Fortification of salt with iodine began in the United States in 1924, and international efforts to iodize salt in the world’s poorest nations continue today.

Dietary iodine deficiency can result in the impaired ability to synthesize T3 and

T4, leading to a variety of severe disorders. When T3 and T4 cannot be produced, TSH is secreted in increasing amounts. As a result of this hyperstimulation, thyroglobulin accumulates in the thyroid gland follicles, increasing their deposits of colloid. The accumulation of colloid increases the overall size of the thyroid gland, a condition called a goiter (Figure 3). A goiter is only a visible indication of the deficiency. Other iodine deficiency disorders include impaired growth and development, decreased fertility, and prenatal and infant death. Moreover, iodine deficiency is the primary cause of preventable mental retardation worldwide. Neonatal hypothyroidism (cretinism) is characterized

Page | 40 DSC1DT: Sensory Physiology, Endocrine and Reproductive System, Renal Physiology

Compiled and circulated by Dr. Parimal Dua, Assistant Professor, Dept. of Physiology, Narajole Raj college by cognitive deficits, short stature, and sometimes deafness and muteness in children and adults born to mothers who were iodine-deficient during pregnancy.

Figure: Goiter.

Hypothyroidism: In areas of the world with access to iodized salt, dietary deficiency is rare. Instead, inflammation of the thyroid gland is the more common cause of low blood levels of thyroid hormones. The condition is called hypothyroidism. Hypothyroidism is characterized by a low metabolic rate, weight gain, cold extremities, constipation, reduced libido, menstrual irregularities, and reduced mental activity. Hyperthyroidism: Hyperthyroidism is an abnormally elevated blood level of thyroid hormones. It is often caused by a pituitary or thyroid tumor. In Graves’ disease, the hyperthyroid state results from an autoimmune reaction in which antibodies overstimulate the follicle cells of the thyroid gland. Hyperthyroidism can lead to an increased metabolic rate, excessive body heat and sweating, diarrhea, weight loss, tremors, and increased heart rate. The person’s eyes may bulge (called exophthalmos) as antibodies produce inflammation in the soft tissues of the orbits. The person may also develop a goiter. 18. Identify the hormone produced by the parafollicular cells of the thyroid. The thyroid gland also secretes a hormone called calcitonin that is produced by the parafollicular cells (also called C cells) that stud the tissue between distinct follicles. Calcitonin is released in response to a rise in blood calcium levels. It appears to have a function in decreasing blood calcium concentrations by:

Page | 41 DSC1DT: Sensory Physiology, Endocrine and Reproductive System, Renal Physiology

Compiled and circulated by Dr. Parimal Dua, Assistant Professor, Dept. of Physiology, Narajole Raj college

 Inhibiting the activity of osteoclasts, bone cells that release calcium into the circulation by degrading bone matrix  Increasing osteoblastic activity  Decreasing calcium absorption in the intestines  Increasing calcium loss in the urine However, these functions are usually not significant in maintaining calcium homeostasis, so the importance of calcitonin is not entirely understood. Pharmaceutical preparations of calcitonin are sometimes prescribed to reduce osteoclast activity in people with osteoporosis and to reduce the degradation of cartilage in people with osteoarthritis. The hormones secreted by thyroid are summarized in Table 1. Thyroid Hormones (Table 1)

Associated hormones Chemical/Class Effect

Thyroxine (T4), Amine Stimulate basal metabolic rate triiodothyronine (T3)

Calcitonin Peptide Reduces blood Ca2+ levels

Of course, calcium is critical for many other biological processes. It is a second messenger in many signaling pathways, and is essential for muscle contraction, nerve impulse transmission, and blood clotting. Given these roles, it is not surprising that blood calcium levels are tightly regulated by the endocrine system. The organs involved in the regulation are the parathyroid glands.

Page | 42 DSC1DT: Sensory Physiology, Endocrine and Reproductive System, Renal Physiology

Compiled and circulated by Dr. Parimal Dua, Assistant Professor, Dept. of Physiology, Narajole Raj college Parathyroid gland: 19. Describe the location and anatomy of the Parathyroid gland. The parathyroid glands are small endocrine glands located in the anterior neck. In another way, the parathyroid glands are located on the posterior, medial aspect of each lobe of the thyroid gland. Anatomically, the glands can be divided into two pairs:  Superior parathyroid glands – Derived embryologically from the fourth pharyngeal pouch. They are usually located at the level of the inferior border of the cricoid cartilage.  Inferior parathyroid glands – Derived embryologically from the third pharyngeal pouch. They are usually located near the inferior poles of the thyroid gland. However in 1-5% of people they can be found deep in the superior mediastinum. (2020)

Figure: Anatomical location of the parathyroid glands

20. Describe the histological structure of Parathyroid gland. There are two types of cells within the parathyroid gland, the chief cells and the oxyphil cells.  Chief cells– The role of this cell type is to secrete parathyroid hormone. They contain prominent Golgi apparatus and endoplasmic reticulum to allow for the synthesis and secretion of parathyroid hormone. The chief cells are the smaller of the two cell types, however they are more abundant.

Page | 43 DSC1DT: Sensory Physiology, Endocrine and Reproductive System, Renal Physiology

Compiled and circulated by Dr. Parimal Dua, Assistant Professor, Dept. of Physiology, Narajole Raj college  Oxyphil cells– These cells are much larger but less abundant than chief cells. Their purpose is unknown. It is interesting to note however that the number of oxyphil cells increases with age and few are seen before puberty.  Histological fat cells (adipose cells) are also seen within the parathyroid gland.

Figure: Anatomical location of the Parathyroid glands and their histology.

21. Discuss the synthesis and release of Parathyroid Hormone. The synthesis of PTH begins within the rough endoplasmic reticulum, where pre-pro-PTH is produced. Pre-pro-PTH is 115 amino acids long and consists of a biologically active sequence, a C terminal fragment sequence, a pro sequence and a signal sequence. The signal sequence is cleaved within the lumen of the endoplasmic reticulum, leaving pro-PTH. After transfer to the Golgi apparatus the pro sequence is also cleaved, resulting in the production of mature PTH, which can then be stored in secretory granules for release. 22. Explain the role of Parathyroid hormones in the balance of calcium in our body. Parathyroid Hormone Actions Parathyroid hormone (PTH) has three main actions, all of which act to increase calcium levels in the body; 1) Increased bone resorption: PTH acts directly on bone to increase bone resorption. It induces cytokine secretion from osteoblasts that act on

Page | 44 DSC1DT: Sensory Physiology, Endocrine and Reproductive System, Renal Physiology

Compiled and circulated by Dr. Parimal Dua, Assistant Professor, Dept. of Physiology, Narajole Raj college osteoclast cells to increase their activity. Osteoclasts are responsible for the breakdown of bone and thus an increase in their activity leads to increased bone break down. This leads to an increase in calcium in the extracellular fluid. PTH also inhibits osteoblasts, the cells involved in bone deposition, thereby sparing blood calcium. 2) Increased reabsorption in the kidney: PTH increases the amount of calcium absorbed from the Loop of Henle and distal tubules, however the mechanism is not fully understood. In addition to this PTH increases the rate of phosphate excretion which is very important to prevent to formation of calcium phosphate kidney stones. 3) Vitamin D synthesis: Although PTH does not actively increase the absorption of calcium from the gut. PTH initiates the production of the steroid hormone calcitriol (also known as 1,25-dihydroxyvitamin D), which

is the active form of vitamin D3, in the kidneys. Calcitriol then stimulates increased absorption of dietary calcium by the intestines. stimulates the formation of vitamin D, this subsequently increases absorption from the gut.

Page | 45 DSC1DT: Sensory Physiology, Endocrine and Reproductive System, Renal Physiology

Compiled and circulated by Dr. Parimal Dua, Assistant Professor, Dept. of Physiology, Narajole Raj college

Figure: Parathyroid Hormone in Maintaining Blood Calcium Homeostasis. Parathyroid hormone increases blood calcium levels when they drop too low. Conversely, calcitonin, which is released from the thyroid gland, decreases blood calcium levels when they become too high. These two mechanisms constantly maintain blood calcium concentration at homeostasis.

Page | 46 DSC1DT: Sensory Physiology, Endocrine and Reproductive System, Renal Physiology

Compiled and circulated by Dr. Parimal Dua, Assistant Professor, Dept. of Physiology, Narajole Raj college 23. Explain the regulation of the synthesis of Parathyroid hormones. Like most endocrine organs the parathyroid gland is controlled by a negative feedback loop. Chief cells have a unique G-protein calcium receptor (CaR) on their surface, which regulates this. When calcium levels in the blood are elevated PTH production must be stopped in order to prevent further elevation of calcium which could lead to hypercalcaemia. Calcium binds to the G protein CaR which subsequently leads to the production of a molecule called phosphoinositide. The activation of this molecule prevents PTH secretion thus calcium is deposited back into the bones. Furthermore as mentioned above PTH stimulates vitamin D synthesis. Vitamin D also acts directly on the parathyroid gland to decrease the transcription of the PTH gene hence less PTH is synthesised. When Calcium is reduced the reverse occurs. Lowered calcium means reduced stimulation of CaR and decreased phosphoinositide. Subsequently PTH secretion is not inhibited. Decreased Vitamin D results in upregulation of PTH gene transcription thus more PTH is synthesised. Note: Elevated phosphate lowers free Calcium in the blood and inhibits the formation of Vitamin D. 24. Discuss the disorders of Hypoparathyroidism, and Hyperparathyroidism. Hyperparathyroidism: Hyperparathyroidism is over-activity of the parathyroid glands and can be classed as primary, secondary, tertiary or malignant depending on the underlying cause. 1) Primary hyperthyroidism is a result of direct alterations to the parathyroid gland such as a benign tumour, hyperplasia or very rarely parathyroid cancer. The excess secretion of PTH leads to elevated calcium in the blood which can cause signs of hypercalcaemia, osteoporosis, osteitis fibrosa cystica and hypertension. 2) Secondary hyperparathyroidism is a physiologically elevated PTH due to reduced calcium levels. This could be caused by chronic renal failure or decreased vitamin D intake. 3) Tertiary hyperparathyroidism occurs after prolonged secondary hyperparathyroidism. This is due to structural changes seen within the gland. To distinguish between secondary and tertiary hyperparathyroidism a blood test will be carried out. Elevated Calcium levels indicate tertiary hyperparathyroidism.

Page | 47 DSC1DT: Sensory Physiology, Endocrine and Reproductive System, Renal Physiology

Compiled and circulated by Dr. Parimal Dua, Assistant Professor, Dept. of Physiology, Narajole Raj college 4) Malignant hyperparathyroidism: some tumours, such as bronchial squamous cell carcinomas, produce a protein called Parathyroid related protein (PTHrP). PTHrP can mimic PTH due to the similarity in their structure which ultimately results in elevated calcium in the blood. However PTH will be reduced due to negative feedback to the parathyroid gland itself.

Figure: X-Ray showing the ‘Salt and Pepper’ sign commonly seen in hyperparathyroidism. It consists of multiple well-defined lucencies, caused by increased bone resorption.

Hypoparathyroidism: Hypoparathyroidism is underactivity of the parathyroid gland and can be classed as primary or secondary depending on the cause. 1) Primary hypoparathyroidism is as a result of decreased PTH secretion due to gland failure. This results in symptoms of hypocalcaemia and patients will often need Calcium supplementation. 2) Secondary hypoparathyroidism is commonly caused by surgical removal of the parathyroid glands. This is often accidental due to the fact that the inferior parathyroid glands are difficult to locate as mentioned above. 25. What is tetany ? Tetany is a symptom characterized by muscle cramps, spasms or tremors. These repetitive actions of the muscles happen when your muscle contracts uncontrollably. Tetany is caused by malfunction of the parathyroid glands and a consequent deficiency of calcium (hypocalcemia). Tetany can also be caused by magnesium deficiency or too little potassium. Having too much acid (acidosis) or too much alkali (alkalosis) in the body can also result in tetany. What brings on these imbalances is another matter altogether. Sometimes kidney failure or problems with the pancreas can interfere with

Page | 48 DSC1DT: Sensory Physiology, Endocrine and Reproductive System, Renal Physiology

Compiled and circulated by Dr. Parimal Dua, Assistant Professor, Dept. of Physiology, Narajole Raj college calcium levels in the body. In these cases, it’s organ failure that leads to tetany by hypocalcemia. Low blood protein, septic shock, and some blood transfusions can also adversely affect blood calcium levels. Sometimes toxins can cause tetany. One example is the botulinum toxin found in spoiled foods or bacteria in soil that enter the body through cuts or injuries. Treatment of tetany includes supplementing with calcium or magnesium, for example. Injecting calcium directly into the bloodstream is the most common approach. However, taking calcium orally (along with vitamin D, for absorption) may be required to prevent it from reoccurring. In some cases, such as kidney failure, ongoing treatment with calcium supplements may be required to treat the condition that led to the tetany.

Page | 49 DSC1DT: Sensory Physiology, Endocrine and Reproductive System, Renal Physiology

Compiled and circulated by Dr. Parimal Dua, Assistant Professor, Dept. of Physiology, Narajole Raj college Adrenal gland: 26. Location and histological structure of adrenal gland. Adrenal Glands have a flattened triangular shape and are embedded in the perirenal fat on the top of the kidneys. The adrenal glands are covered with a thick fibrous connective tissue capsule from which trabeculae extend into the parenchyma, carrying blood vessels and nerves.

Each adrenal gland has two parts external adrenal cortex and internal adrenal medulla. Both adrenal cortex and medulla have different embryonic origin, structure and functions. Adrenal Cortex: The adrenal cortex is derived from the mesoderm of the embryo. The adrenal cortex is the steroid secreting portion. It lies beneath the capsule and constitutes nearly 90% of the gland by weight. The adrenal cortex is subdivided into three zones on the basis of the arrangement of its cells and these three concentric zones can be distinguished in the cortex: 1) Zona glomerulosa - thin, outermost zone 2) Zona fasiculata - thick, middle zone 3) Zona reticularis - thin, inner zone 1) Zona glomerulosa (glomerul means little ball): This is the outer zone that lies just below the capsule. It constitutes about 15% of the gland. Its cells are closely packed and arranged in spherical clusters and arched columns which secrete hormones called mineralocorticoids because they affect mineral homeostasis.

Page | 50 DSC1DT: Sensory Physiology, Endocrine and Reproductive System, Renal Physiology

Compiled and circulated by Dr. Parimal Dua, Assistant Professor, Dept. of Physiology, Narajole Raj college 2) Zona fasciculata (fascicul means little bundle): This is the middle zone which is widest of the three zones. It constitutes about 50% of the gland. Cells in the fasiculata are polyhedral and usually have a foamy appearance due to abundant lipid droplets. They also are arranged in long, straight columns that radiate toward the medulla. The cells of this zone secrete mainly glucocorticoids, which are named because they affect glucose homeostasis. 3) Zona reticularis (reticul means network): This is the inner zone that constitutes about 7% of the gland. The cells within this zone are arranged in branching cords that project in many different directions and anastomose with one another. These cells secrete gonadocorticoids (e.g., androgens that have masculinizing effects). The cells of the zona fasciculata and zona reticulata contain ascorbic acid (vitamin C). Adrenal Medulla: Adrenal Medulla originates from neural crest cells (neuroectoderm) of the embryo. Adrenal Medulla is the catecholamine secreting potion. It lies deep to the cortex and forms the center of the gland. Adrenal Medulla is composed of a parenchyma of large, pale-staining epithelioid cells called chromaffin cells, connective tissue, numerous sinusoidal blood capillaries and nerves. Ganglion cells are also present in the medulla. Chromaffin cells are modified postganglionic cells of sympathetic nervous system which have lost normal processes and have acquired a glandular function. These cells are connected with the preganglionic motor fibres of the sympathetic nervous system. Obviously, the adrenal medulla is simply an extension of the sympathetic nervous system. The medulla of the adrenal glands secretes two hormones: 1) Norepinephrine (noradrenaline) and 2) Epinephrine (adrenaline). Norepinephrine and epinephrine are derived from tyrosine amino acid.

Page | 51 DSC1DT: Sensory Physiology, Endocrine and Reproductive System, Renal Physiology

Compiled and circulated by Dr. Parimal Dua, Assistant Professor, Dept. of Physiology, Narajole Raj college

Histologic examination of the adrenal gland reveals a rich vasculature. Numerous small arteries from several sources ramify over the surface of the gland and penetrate into the gland into two ways:  Cortical arteries and arterioles branch into capillary beds within the cortex to supply that area then coalesce into veins at the corticomedullary junction.  Medullary arteries and arterioles penetrate the cortex without branching, then form capillary beds in the medulla. Blood from both cortical and medullary veins empties through a single large central vein which leaves the adrenal gland and anastomoses with either the vena cava or renal vein.

Page | 52 DSC1DT: Sensory Physiology, Endocrine and Reproductive System, Renal Physiology

Compiled and circulated by Dr. Parimal Dua, Assistant Professor, Dept. of Physiology, Narajole Raj college 27. Discuss the synthesis and release of adrenal gland. Adrenal cortex produces several steroid hormones and adrenal medulla produces catecholamines as depicted in the table below. All hormones of adrenal cortex are synthesized from cholesterol. There is some overlap in hormones synthesized by the zonae fasiculata and reticularis (i.e. cells in the fasiculata produce a small amount of androgens and cells in the reticularis secrete some cortisol). Parts of zones of the gland Name of the hormones Adrenal Gland Cortex Zona glomerulosa Mineralocorticoids (aldosterone) Zona fasiculata Glucocorticoids (cortisone, cortisol), androgens Zona reticularis Sex steroids (androgens), cortisol Medulla Catecholamines (epinephrine and norepinephrine)

28. Write down the functions of different hormones of adrenal gland. Hormones of adrenal cortex: Corticosteroids (corticoids—hormones of adrenal cortex) are grouped into three categories : mineralocorticoids, glucocorticoids and gonad corticoids. A. Mineralocorticoids: These hormones are secreted by the cells of zona glomerulosa of adrenal cortex. As the name indicates, they are responsible for the regulation of mineral metabolism. Aldosterone (salt-retaining hormone) is the principal mineralocorticoid (90 to 95%) in humans. 1) On Mineral Metabolism: i. Mineralocorticoids help to increase the rate of tubular reabsorption of sodium (Na+) from the renal tubules. ii. It increases the excretion of K+ from distal convoluted tubule and collecting tubule to maintain the K+ concentration. iii. Helps in retention of NaCI. iv. Increased secretion may cause alkalosis while decreased secretion promotes acidosis.

Page | 53 DSC1DT: Sensory Physiology, Endocrine and Reproductive System, Renal Physiology

Compiled and circulated by Dr. Parimal Dua, Assistant Professor, Dept. of Physiology, Narajole Raj college 2) On Water Metabolism: Helps in absorption of water from renal tubules to maintain the water balance. 3) On Blood Protein: Helps in increase of haemoglobin and plasma protein. 4) On Fat: Stimulates fatty acid synthesis. 5) On Cardiovascular System: i. Hyper-secretion increases the cardiac output, while hypo-secretion re- duces the cardiac output. ii. Excess secretion increases the arterial blood pressure. 6) On Fluid Volume: i. This hormone stimulates the thirst centre and as a result person shows thirsty. ii. Helps to increase the plasma volume. 7) On Cellular Mechanism: Helps in diffusion of lipid in cellular membranes of tubular epithelial cells. B. Glucocorticoids: As their name suggests, they affect carbohydrate metabolism, however, they also affect the metabolism of proteins and fats. Glucocorticoids include three main hormones: cortisol (= hydrocortisone), corticosterone and cortisone. 1) On Carbohydrate Metabolism: i. Promotes gluconeogenesis (synthesize carbohydrates from non- carbohydrates such as amino acids and glycerol) in liver especially from proteins to increase blood glucose level. ii. Stimulates glycogenesis in liver and muscle. iii. Helps in phosphorylation of glucose- 6-P. iv. Helps in absorption of glucose from intestine and renal tubule. 2) On Protein Metabolism: i. It stimulates protein synthesis in liver, but reduces protein synthesis in muscle. ii. Glucocorticoids stimulates the degradation of proteins within cells and inhibit amino acid uptake in extra-hepatic cells. iii. It depresses amino acids transport into muscle cells. iv. It enhances transport of amino acids in hepatic cells. v. It promotes urea synthesis in liver in presence of arginase and argino suc- cinate synthetase.

Page | 54 DSC1DT: Sensory Physiology, Endocrine and Reproductive System, Renal Physiology

Compiled and circulated by Dr. Parimal Dua, Assistant Professor, Dept. of Physiology, Narajole Raj college 3) On Fat Metabolism: i. Glucocorticoids stimulate the break-down of fats in adipose tissue and may raise free fatty acids in plasma. Thus cortisol has anti insulin effect. ii. It stimulates the synthesis of triglycerides in the liver. iii. Helps in metabolism of fatty acids to ketone bodies. iv. Stimulates fat absorption from the intestine. v. It stimulates the mobilization of fatty acids and glycerol from adipose tissue in the blood. 4) On GI System: i. It stimulates oxyntic cells of stomach for secretion of HCI. ii. It stimulates secretion of trypsinogen and pepsinogen. iii. It inhibits absorption of calcium from small intestine. 5) On Nervous System: i. It influences the survival neurons. ii. Maintains the structural integrity of brain. iii. Excess secretion leads to reduction in threshold of electrical stimulation of brain which may cause convulsion. 6) On Ca++ metabolism: i. It reduces intestinal Ca++ absorption. ii. It increases urinary Ca++ excretion. 7) On Respiratory System: It causes maturation of foetal lung. 8) Effects on Cardiovascular System: i. It stimulates cardiac output. ii. It increases blood pressure by enhancing the action of catecholamine’s. iii. It reduces capillary permeability. 9) On Muscular System: It causes loss of muscle power. 10) On Inflammatory and Immunologic Effects: i. It inhibits inflammatory and allergic reactions. ii. It retards phagocytic activities of WBCs and thus suppresses ‘inflammation reaction’. iii. This hormone decreases the WBC count of blood. iv. Decreases the number of circulating monocytes, eosinophil’s and lymphocytes. v. Cortisol suppresses synthesis of antibodies by inhibiting the production of lymphocytes in the lymphoid tissues. vi. Reduces the migration of leucocytes into affected tissue.

Page | 55 DSC1DT: Sensory Physiology, Endocrine and Reproductive System, Renal Physiology

Compiled and circulated by Dr. Parimal Dua, Assistant Professor, Dept. of Physiology, Narajole Raj college vii. This hormone also reduces the number of mast cells, reducing secretion of histamine from mast cells. Cortisol is used for treatment of allergy. viii. Inhibits the synthesis of IL-1. ix. It is also used in transplantation surgery to suppress the formation of antibodies in the body of recipients so that the latter may accept the transplanted organs. 11) On RBCs: It increases red blood cells count. Cortisol has the capacity to cope with stress. When we are under stress our body secretes cortisol that is why this hormone is called “stress hormone”. C. Gonad corticoids (Sex-corticoids): They are also called sex hormones of adrenal glands. Large quantities of male than female sex-corticoids (sex hormones) are produced. These male sex hormones are called androgens which are important in the development of a male foetus. 1) Effects in Males: i. Androgens stimulate the development of male secondary sexual characters like distribution of body hair including pubic and axillary hair. It may influence the growth and differentiation of secondary sex organs. ii. The adult individual can cause opposite sex attraction, coarsening of the voice. iii. Helps is growth of muscular, growth of sex glands and increases BMR. 2) Effects in Females: i. Female sex hormones secreted by the adrenal cortex are oestrogens which maintain the development of female secondary sexual characters.These hormones may be responsible for libido in women. ii. Overproduction may cause hirsutism and virilization. iii. Hyper-secretion may cause involution of female secondary sex char- acters. iv. May cause development of mammary gland, more subcutaneous fat, less body hair and feminine voice. The adrenal cortex is essential for life. Its removal or destruction is fatal unless the hormones produced by it are supplemented artificially. Hormones of Adrenal Medulla: The medulla of the adrenal glands secretes two hormones: norepinephrine (noradrenaline) and epinephrine (adrenaline). Norepinephrine and epinephrine are derived from tyrosine amino acid.

Page | 56 DSC1DT: Sensory Physiology, Endocrine and Reproductive System, Renal Physiology

Compiled and circulated by Dr. Parimal Dua, Assistant Professor, Dept. of Physiology, Narajole Raj college 1) Norepinephrine (= Noradrenaline): It regulates the blood pressure under normal condition. It causes constriction of essentially all the blood vessels of the body. It causes increased activity of the heart, inhibition of gastrointestinal tract, dilation of the pupils of the eyes and so forth. 2) Epinephrine (= Adrenaline): It is secreted at the time of emergency. Hence it is also called emergency hormone. It causes almost the same effects as those caused by norepinephrine, but the effects differ in the following respects: i. First, epinephrine has a greater effect on cardiac activity than norepinephrine. ii. Second, epinephrine causes only weak constriction of the blood vessels of the muscles in comparison with a much stronger constriction that results from norepinephrine. iii. A third difference between the action of epinephrine and norepinephrine relates to their effects on tissue metabolism. Epinephrine probably has several times as great a metabolic effect as norepinephrine. Target Cells: Both adrenaline and noradrenaline acts on the cells of skeletal, cardiac and smooth muscles and blood vessels and fat cells. Because of the role of their hormones, the adrenal glands are also called ‘glands of emergency’. Sympatheticoadrenal System: Stimulation of the sympathetic nerves to adrenal medulla causes large quantities of epinephrine (adrenaline) and norepinephrine (noradrenaline) to be released into the blood circulation and then these hormones are carried to all the tissues of the body. Both the hormones (epinephrine and norepinephrine) and sympathetic nervous system act on the same organs and produce similar effects on them (e.g., accelerates heart beat, raises blood pressure, slows peristalsis, etc.). Since the sympathetic nervous system and the adrenal medulla function as an integrated system, it is called sympatheticoadrenal system. Adrenaline hormone is responsible for “fight or flight response”. 29. Describe the hypo and hyper-active states of adrenal cortex. Hypo-active states of adrenal cortex: 1) Addison’s disease:

Page | 57 DSC1DT: Sensory Physiology, Endocrine and Reproductive System, Renal Physiology

Compiled and circulated by Dr. Parimal Dua, Assistant Professor, Dept. of Physiology, Narajole Raj college This disease is caused by the deficiency of mineralocorticoids and glucocorticoids. It is also caused by the destruction of adrenal cortex in disease such as tuberculosis. Its symptoms include low blood sugar, low plasma Na+, high K+ plasma, increased urinary Na+, nausea, vomiting, diarrhoea and a bronze-like pigmentation of skin. Severe dehydration is also common in the person suffering from this disease. Hyper-active states of adrenal cortex: 1) Cushing’s Syndrome (Fig. 22.13): It is caused by excess of cortisol which may be due to a tumour of the adrenal cortex. It is characterised by high blood sugar, appearance of sugar in the urine, rise in plasma Na+, fall in plasma K+, rise in blood volume, high blood pressure, obesity and wasting of muscles of thighs and pectoral and pelvic girdles.

2) Aldosteronism (Conn’s Syndrome): Excessive production of aldosterone from an adrenal cortical tumour causes this disease. Its symptoms include a high plasma Na+, low plasma K+, rise in blood volume, high blood pressure and polyurea. 3) Adrenal Virilism (Fig. 22.13): Appearance of male characters in female is called virilism. Excessive production of male sex-corticoids (androgens) produces male secondary sexual characters like beard, moustache, hoarse voice in woman.

Page | 58 DSC1DT: Sensory Physiology, Endocrine and Reproductive System, Renal Physiology

Compiled and circulated by Dr. Parimal Dua, Assistant Professor, Dept. of Physiology, Narajole Raj college 4) Gynaecomastia: It is the development of enlarged mammary glands (breasts) in the males. It is due to excessive secretion of female sex hormones (oestrogens) in males. Decreased testosterone may also lead to gynaecomastia. 30. Can chomaffin cells be found outside of the adrenal medulla ? Yes. A rare tumor derived from chromaffin cells. It produces excessive amounts of catecholamines. Most pheochromocytomas contain predominantly chomaffin cells that secrete norepinephrine in comparison with the normal adrenal medulla that comprises about 85% epinephrine-secreting cells. 31. Write the differences between Adrenal Cortex and Adrenal Medulla of Humans. Adrenal Cortex Adrenal Medulla 1) It is outer firm part of the adrenal It is central soft part of the adrenal gland. gland. 2) It forms about 75% part of the gland. It forms about 25% of the gland. 3) It is enclosed by a fibrous capsule. It is not enclosed by a fibrous capsule. 4) It develops from the mesoderm. It develops from ectoderm. 5) It comprises three regions or zones: It is not differentiated into regions. It (a) Outer thin zona glomeruiosa. consists of chromophil cells. The (b) Middle thick zona fasciculata and adrenal medulla is simply an (c) Inner thin zona reticularis. extension of the sympathetic nervous system. 6) It is essential for life. Its destruction It is not so essential for life. Its causes death. destruction does not cause death. 7) It is stimulated to release its It is stimulated to secrete its hormone hormones by adrenocorticotrophic by nerve impulses reaching through hormone (ACTH) from the anterior sympathetic nerve fibres. lobe of pituitary gland. 8) It secretes three groups of hormones: It secretes two similar hormones: mineralocorticoids glucocorticoids noradrenaline and adrenaline. and gonadocorticoides. 9) There is no cooperation between Adrenal medulla and sympathetic adrenal cortex and sympathetic nervous system function as an nervous system. integrated system called sympatheticoadrenal system.

Page | 59 DSC1DT: Sensory Physiology, Endocrine and Reproductive System, Renal Physiology

Compiled and circulated by Dr. Parimal Dua, Assistant Professor, Dept. of Physiology, Narajole Raj college 32. Describe the biosynthesis of adrenocortical hormone. All adrenocortical hormones, known as Corticosteroids, are steroid hormones. As with all steroid hormones, biosynthesis of corticosteroids is based on sequential enzymatic modification of cholesterol. The three different subtypes of adrenocortical hormones are achieved by different pathways of enzymatic modification, yielding hormones with different chemical side groups. Because each zone of the adrenal cortex expresses a different set of modifying enzymes, different hormones are produced in each zone. Each zone of the adrenal cortex possesses only those enzymes necessary to synthesize its particular hormone; thus, no zone possesses all of the enzymes shown. The only enzyme which is shared by all the zones is Cholesterol Desmolase which is required to activate cholesterol for any further chemical modification. Expression of cholesterol desmolase depends on the presence of ACTH a hypothalamic hormone; therefore, in the absence of ACTH no adrenocortical hormones are produced. The synthesis of adrenocortical hormones occurs in following steps: 1) Conversion of cholesterol to pregnenolone: The adrenal steroid hormones are synthesized from precursor cholesterol molecule. Cholesterol is mostly derived from plasma, but a small portion is synthesized in situ from acetyl CoA. Much of the cholesterol in the adrenal is esterified and steroid in cytoplasmic lipid droplets. Upon stimulation of the adrenal by ACTH (or cAMP), an esterase is activated, and the free cholesterol formed is transported into the

mitochondrion, where cytochrome P450 side chain cleavage enzyme (P450 sec) converts cholesterol to pregnenolone. Pregnenolone acts as a pivot for synthesis of all adrenocorticosteroids. Pregnenolone is then released from the mitochondria and enter into the smooth ER. 2) Pregnenolone to different adrenocorticosteroids: Pregnenolone leads to the synthesis of three hormones by successive reactions. (A) Mineralocorticoid synthesis or aldosterone synthesis: In the zona glomerulosa, pregnenolone is converted to progesterone by the action two smooth endoplasmic reticulum enzymes 3β-OHSD and ∆5, 4 isomerase. Progesterone is then hydroxylated at C21 position to form 11- deoxycorticosterone by the actions of microsomal enzyme 21-hydroxylase. 11-deoxycorticosterone is transferred into mitochondria where next

hydroxylation at C11 position, produces corticosterone by the action of 11 β-

Page | 60 DSC1DT: Sensory Physiology, Endocrine and Reproductive System, Renal Physiology

Compiled and circulated by Dr. Parimal Dua, Assistant Professor, Dept. of Physiology, Narajole Raj college hydroxylase. The 18-hydroxylase acts on corticosterone to form 18- hydroxycorticosterone. 18-hydroxycorticosterone has its C18 hydroxyl group oxidized to an aldehyde groups by 18-hydroxysteroid dehydrogenase, to be changed into the aldosterone. (B) Glucocorticoids synthesis: In the zona fasiculata cells during glucocorticoid synthesis cholesterol is converted to pregnenolone like mineralocorticoid synthesis. Pregnenolone follows Cortisol synthesis in one hand and corticosterone synthesis on other hand. Cortisol synthesis requires three hydroxylases that act sequentially on

C17 C21 and C11 positions. The first two reactions are rapid, while

C11 hydroxylation is relatively slow. Pregnenolone is changed successively through 1 7-hydroxypregnenolone and 1 7-hydroxyprogesterone to 11- deoxycortisol by the actions of microsomal enzymes—1 7α-hydroxylase, 3β- OHSD, ∆5,4 isomerase and 21 hydroxylase respectively. 11 -deoxycortisol is then trans-located to mitochondria, where it is hydroxylated by mitochondrial enzyme 11 β-hydroxylase to produce Cortisol which is the most potent glucocorticoid hormone in humans. A small amount of pregnenolone on other hand gives rise to another glucocorticoid hormone corticosterone. Corticosterone biosynthesis takes place through progesterone like aldosterone pathway. (C) Sex steroid synthesis: The zona reticularis cells mainly take part in this synthesis. In adrenal sex steroid synthesis, basic pregnenolone formation takes place from cholesterol like other adrenocorticosteroid synthesis. The C17-side chain of small amounts of 1 7-hydroxypregnenolone, is oxidatively cleaved away and give rise to dehydroepiandrosterone (DHEA) under the action of microsomal 1 7, 20-Lyase. Most of DHEA is rapidly modified by the addition of sulfate, about half of which occurs in the adrenal and the rest in the liver. DHEA sulfate is inactive, but removal of the sulfate results in reactivation. 3β-OHSD and ∆5,4 isomerase convert DHEA into more potent sex steroid androstenedione (Fig. 6.17).

Page | 61 DSC1DT: Sensory Physiology, Endocrine and Reproductive System, Renal Physiology

Compiled and circulated by Dr. Parimal Dua, Assistant Professor, Dept. of Physiology, Narajole Raj college

Page | 62 DSC1DT: Sensory Physiology, Endocrine and Reproductive System, Renal Physiology

Compiled and circulated by Dr. Parimal Dua, Assistant Professor, Dept. of Physiology, Narajole Raj college

Figure: Adrenal steroid biosynthesis involves the shuttling of precursors between mitochondria and the ER. 33. How to control the synthesis of adrenocortical steroid hormones ? The synthesis and release of adrenocorticosteroids is controlled by pituitary ACTH. Increase in the concentration of glucocorticoids in blood inhibits the secretion of ACTH by negative feedback mechanism. ACTH-RH of hypothalamus controls the secretion of pituitary ACTH. The different factors like stress, cold, excitement, diurnal rhythm etc. control the secretion of hypo- thalamus (Figure 1) Mineralocorticoids are mainly regulated by plasma concentrations of K+ and Na+, renin-angiotensin system.

Page | 63 DSC1DT: Sensory Physiology, Endocrine and Reproductive System, Renal Physiology

Compiled and circulated by Dr. Parimal Dua, Assistant Professor, Dept. of Physiology, Narajole Raj college

Figure: Mechanism of Action of ACTH in the ZF of the Adrenal Cortex

The synthesis of adrenocortical steroid hormones involves a chain of oxidation- reduction reactions catalyzed by a series of enzymes. Synthesis begins with a molecule of cholesterol. Through shared intermediates and pathways branching off those shared intermediates, the different classes of steroids are synthesized. Steroids are synthesized from cholesterol in their respective regions of the adrenal cortex. The process is controlled by steroidogenic acute regulatory protein (StAR) which sits in the mitochondrial membrane and regulates the passage of cholesterol. This is the rate-limiting step of steroid biosynthesis. Once StAR has transported cholesterol into the mitochondria, the cholesterol molecule undergoes a string of oxidation-reduction reactions catalyzed by a series of enzymes from the family of cytochrome P450 enzymes. A coenzyme system called adrenodoxin reductase transfers electrons to

Page | 64 DSC1DT: Sensory Physiology, Endocrine and Reproductive System, Renal Physiology

Compiled and circulated by Dr. Parimal Dua, Assistant Professor, Dept. of Physiology, Narajole Raj college the P450 enzyme which initiates the oxidation-reduction reactions that transform cholesterol into the steroid hormones. Though synthesis is initiated inside mitochondria, precursors are shuttled to the endoplasmic reticulum for processing by enzymes present in the endoplasmic reticulum. The precursors are shuttled back to the mitochondria in the region of the adrenal cortex within which synthesis initially began and it is there that synthesis is completed. 34. How to control the synthesis of Glucocorticoid (Cortisol) hormone ? Cortisol functions to increase blood sugar through gluconeogenesis, to suppress the immune system, and to aid in the metabolism of lipids, proteins, and carbohydrates. Neuroendocrine pathways in emotional stress leading to adrenal activation:

35. Describe the transport and Catabolism of Adrenocortical Hormones. Transport of Adrenocortical Hormones: Glucocorticoids are transported through plasma α-globulin, called transcortin or Corticosteroid Binding Globulin (CBG). Mineralocorticoid does not have a specific plasma transport protein, but it forms a very weak association with albumin. Sex steroid is transported through plasma protein. Catabolism of Adrenocortical Hormones: Mineralocorticoid is rapidly cleared from the plasma by the liver. The liver forms tetrahydroaldosterone glucuronide which is excreted via urine. Androstenedione and DHEA are

Page | 65 DSC1DT: Sensory Physiology, Endocrine and Reproductive System, Renal Physiology

Compiled and circulated by Dr. Parimal Dua, Assistant Professor, Dept. of Physiology, Narajole Raj college excreted as 17-keto compounds. Glucocorticoids are metabolised in the liver and excreted in urine as conjugates with glucuronide or sulfates. 36. Write the short notes on Pheochromocytoma and Neuroblastoma. There are two tumours of the adrenal medulla; they are uncommon but very important. Pheochromocytoma: Pheochromocytoma is usually benign, in that it enlarges but does not spread to distant sites. Although benign, it produces severe symptoms and may be fatal. This is because it is derived from cells that produce epinephrine (adrenaline) and norepinephrine (noradrenaline), and the tumours produce these active chemicals in excessive amounts, leading to uncontrollable high blood pressure, which may be fatally complicated by a stroke or severe heart failure. Neuroblastoma: Neuroblastoma is a highly malignant tumour of the adrenal medulla which is derived from primitive neural crest embryonic stem cells (neuroblasts), from which the adrenal medulla develops in embryonic life. Occasionally, some of these cells persist after birth and may develop into this tumour. This is an example of a so-called ‘embryonal’ tumour; another type is seen in the kidney in children. 37. How to control the synthesis of adrenomedullary hormones ? The secretion of medullary epinephrine and norepinephrine is controlled by a neural pathway that originates from the hypothalamus in response to danger or stress (the sympathomedullary pathway). The sympathomedullary (SAM) pathway involves the stimulation of the medulla by impulses from the hypothalamus via neurons from the thoracic spinal cord. One of the major functions of the adrenal medulla is to respond to stress. Stress can be either physical or psychological or both. Physical stresses include exposing the body to injury, walking outside in cold and wet conditions without a coat on, or malnutrition. Psychological stresses include the perception of a physical threat, a fight with a loved one, or just a bad day at school. The body responds in different ways to short-term stress and long-term stress following a pattern known as the general adaptation syndrome (GAS). Stage one of GAS is called the alarm reaction. This is short-term stress, the fight-or- flight response, mediated by the hormones epinephrine and norepinephrine from the adrenal medulla via the SAM pathway. Their function is to prepare the body for extreme physical exertion. Once this stress is relieved, the body quickly returns to normal.

Page | 66 DSC1DT: Sensory Physiology, Endocrine and Reproductive System, Renal Physiology

Compiled and circulated by Dr. Parimal Dua, Assistant Professor, Dept. of Physiology, Narajole Raj college If the stress is not soon relieved, the body adapts to the stress in the second stage called the stage of resistance. If a person is starving for example, the body may send signals to the gastrointestinal tract to maximize the absorption of nutrients from food. If the stress continues for a longer term however, the body responds with symptoms quite different than the fight-or-flight response. During the stage of exhaustion, individuals may begin to suffer depression, the suppression of their immune response, severe fatigue, or even a fatal heart attack. These symptoms are mediated by the hormones of the adrenal cortex, especially cortisol, released as a result of signals from the HPA axis.

Pancreas:

Page | 67 DSC1DT: Sensory Physiology, Endocrine and Reproductive System, Renal Physiology

Compiled and circulated by Dr. Parimal Dua, Assistant Professor, Dept. of Physiology, Narajole Raj college 38. Describe the location and histological structure of panchreatic islets. Pancreas is derived from the endoderm of the embryo. The pancreas lies inferior to the stomach in a bend of the duodenum. It is both an exocrine and an endocrine gland. A large pancreatic duct runs through the gland, carrying enzymes and other exocrine digestive secretions from the pancreatic acinar cells to the small intestine. The tissue of the pancreas has in addition to the acinar cells, groups of cells called islets of Langerhans, after the name of their discoverer (1869). These produce endocrine secretions. Four kinds of cells have been identified in the islets: 1) Alpha cells or А-cells (about 15%): Alpha cells are usually found towards the periphery of the islet. These cells produce glucagon. 2) Beta cells or В-cells (about 65%): Beta cells are usually found towards the middle of the islet. These cells produce insulin. 3) Delta cells or D-cells (about 5%): Delta cells are found scattered of the islet. These cells produce somatostatin (SS). 4) Pancreatic Polypeptide cells or PP cells or F-cells (15%): F-cells are found scattered of the islet. These cells produce pancreatic polypeptide (PP). Many capillaries will be intersperced between these cells.

Page | 68 DSC1DT: Sensory Physiology, Endocrine and Reproductive System, Renal Physiology

Compiled and circulated by Dr. Parimal Dua, Assistant Professor, Dept. of Physiology, Narajole Raj college 39. Write down the role of different hormones of Pancreas. A. Glucagon: Glucagon acts on the cells of the liver and adipose tissue. It stimulates the liver to convert stored glycogen into glucose. Glucagon is also called an “anti-insulin” hormone. It performs following functions: 1) On carbohydrate metabolism: i. Glucagon is sensitive to the adenylate cyclase receptor sites in the liver and increases cAMP level. cAMP activates the enzyme protein kinase which further activates phosphorylase. Phosphorylase causes glycogenolysis making available of glucose in blood. cAMP suppresses glycogen synthetase. Thus, glucagon increases glycogen breakdown and inhibits synthesis of glycogen. It has no effect on glycogen phosphorylase of muscles. ii. It depresses glycogenesis in the liver by reducing glycogen synthetase activity. iii. Glucagon can also activate various phospho-enzymes by activating protein kinase and inhibit de-phospho-enzymes. Thus, glucagon increases the gluconeogenesis in the liver from proteins by activation of pyruvate carboxylase and fructose-1, 6-di-phosphatase. iv. It also inhibits glucose oxidation by inhibiting pyruvate kinase and pyruvate dehydrogenase. 2) On protein metabolism: i. It reduces protein synthesis by depressing the incorporation of amino acids into peptide chains. ii. It stimulates protein catabolism and thus increases nitrogenous waste metabolites. 3) On fat metabolism: i. In adipose tissue as well as in liver, it increases the breakdown of lipids to fatty acids and glycerol. Therefore, It increases free fatty acids and glycerol in blood by lipolysis. ii. It increases adenyl cyclase activity in adipose tissue that results in in- creased lipolysis. iii. It has ketogenic effects. It increases fatty acid oxidation and ketosis to supply more fatty acids to the liver. 4) On GI tract: i. It increases the volume of secretion by the small intestine.

Page | 69 DSC1DT: Sensory Physiology, Endocrine and Reproductive System, Renal Physiology

Compiled and circulated by Dr. Parimal Dua, Assistant Professor, Dept. of Physiology, Narajole Raj college ii. It may stimulated gastric and pancreatic enzymes. 5) On mineral metabolism: i. It promotes cellular release of potassium, calcium and inorganic phos- phate. ii. It tends to decrease hypercalcaemia and hypophosphatemia by increasing calcitonin. 6) On heart: It has an ionotropic effect on the heart and increases the force of myocardial contraction, cardiac output and arterial blood pressure. 7) Elevated glucagon concentration also increases ketogenesis. 8) Crystalline glucagon polypeptide is used for the treatment of hypoglycemic persons. Glucagon is also used as a diagnostic test for glycogen storage disease. B. Insulin: The following functions are believed to be performed by insulin: 1) On carbohydrate metabolism: a) Stimulates Glycolysis: Insulin increases combustion of sugar in the tissues by inducing glycolysis with the help of pyruvate kinase, glucokinase and phosphofructokinase. b) Stimulates Glycogenesis: Insulin increases synthesis of glycogen from monosaccharides and lactates both in liver and muscles by increasing activities of glycogen synthetase, hexokinase II and a microsomal glucokinase. c) Prevents Gluconeogenesis: Glucose is normally formed from proteins and fats in the liver. It regulates hepatic gluconeogenesis by inducing pyruvate carboxylase, G-6-phosphatase etc. But in diabetes the rate of gluconeogenesis is increased. In starving diabetic, Dextrose/Nitrogen ratio is nearly constant, about 3.6 indicating that both sugar and nitrogen are derived from the same source —the proteins. When insulin is given, both sugar and nitrogen excretion falls, indicating that formation of new glucose from proteins has been stopped. 2) On protein metabolism: a) It stimulates the synthesis of tissue protein by taking amino acids from blood. b) It depresses protein catabolism and reduces the process of gluconeogenesis from protein. 3) On fat metabolism: It increases the synthesis of fat in the adipose tissue from fatty acids. Insulin reduces the breakdown and oxidation of fat.

Page | 70 DSC1DT: Sensory Physiology, Endocrine and Reproductive System, Renal Physiology

Compiled and circulated by Dr. Parimal Dua, Assistant Professor, Dept. of Physiology, Narajole Raj college a) It helps in the accumulation of depot fat i.e. lipogenesis. It produces pyruvic acid from glucose oxidation, the part of the pyruvic acid after decarboxylation is converted to acetic acid which will give rise to neutral fats. b) It increases the activity of lipoprotein lipase in adipose tissues. The enzymes catalyzes hydrolysis of chylomicrons and lipoproteins to yield fatty acids which enter the adipose tissue cells. c) It increases ketone bodies from fatty acids in liver. The presence of ketone bodies can cause severe acidosis and coma in patients. d) It reduces the free fatty acids level in blood by depressing the activity adipose tissue lipase. 4. Anti-Ketogenic Action: It prevents formation of ketone bodies. In the deficiency of the hormone (diabetes) more ketone bodies are formed in the liver due to incomplete combustion of fatty acids. On administration of insulin more sugar burns, and liver glycogen content increases displacing the fats: thus ketone formation is prevented by insulin. C. Somatostatin (SS): Somatostatin acts on the cells of the pancreas. The same substance as growth inhibiting hormone from the hypothalamus, is produced not only by the pancreas and hypothalamus but also by some cells of the digestive tract. One of the actions of somatostatin seems to suppress the release of other hormones from the pancreas. It also appears to suppress the release of hormones from the digestive tract. 1) It may control the peristaltic activity of GI tract. 2) It may regulate insulin and glucagon secretion by paracrine action. 3) It helps in transport of nutrients from GI tract to the circulation. 4) It decreases HCI secretion of stomach in empty state. D. Pancreatic Polypeptide (PP): Pancreatic polypeptide act on the cells of the pancreas. It appears that pancreatic polypeptide inhibits the release of digestive secretion of the pancreas. 40. Discuss the endocrine disorder of the pancreas. 1) Diabetes mellitus (Hyperglycemia): The most common endocrine disorder of the pancreas is the diabetes mellitus, now recognized to exist in two forms — insulin-dependent and non-insulin-dependent.

Page | 71 DSC1DT: Sensory Physiology, Endocrine and Reproductive System, Renal Physiology

Compiled and circulated by Dr. Parimal Dua, Assistant Professor, Dept. of Physiology, Narajole Raj college The insulin-dependent diabetes mellitus (IDDM) is caused by a failure of the Beta-cells to produce adequate amount of insulin while the non-insulin- dependent diabetes mellitus (NIDDM) appears to involve failure of insulin to facilitate the movement of glucose into cells. The effects of the hypo-activity of the insulin are: a) In both disorders the blood glucose concentration is elevated above the normal range (Hyperglycaemia). Some of the glucose is excreted in the urine (glycosuria), and water follows the glucose, causing excessive urination and dehydration of body tissues. This causes excessive thirst (polydipsia). The cells are unable to utilize glucose and other carbohydrates for energy production. b) Depletion of glycogen depots of liver and muscles; lowering of R. Q. (Respiratory Quotient) c) They utilize their proteins for it. The person becomes very weak. Degradation of fats increases, producing ketone bodies (ketosis). The latter are acidic and poisonous. Blood cholesterol level rises. Healing power is impaired. d) The arteriovenous blood sugar difference becomes low indicating that peripheral tissues are not utilising blood sugar. e) This results in weakening the animal body with early onset of muscle fatigue and finally ends in death. Diabetes mellitus is diagnosed by the above characters and the sufferers are temporarily cured by injecting insulin [Protamine zinc insulin —a salt of zinc, basic protein (Protamine) and insulin]. Administration of insulin lowers the blood-glucose level. It gives relief to the patient. A tendency towards non-insulin-dependent diabetes appears to be inherited as an autosomal recessive characteristic. 2) Hypoglycemia: It occurs when the blood glucose level falls below normal. Theoretically, it may be caused by an excess of insulin, a deficiency of glucagon, or a failure of the secretion of the two hormones to completely regulate the blood sugar. Some individuals have been found to have few or no Alpha cells and thus are deficient in glucagon, whereas others produce excess quantities of insulin usually because of a tumour of the beta cells. The presence of excess insulin is more correctly referred to as hyperinsulinism. Symptoms of hypoglycemia include weakness, profuse

Page | 72 DSC1DT: Sensory Physiology, Endocrine and Reproductive System, Renal Physiology

Compiled and circulated by Dr. Parimal Dua, Assistant Professor, Dept. of Physiology, Narajole Raj college sweating, irritability, confusion, unconsciousness and convulsions. It needs urgent intake of sugar or glucose. 41. Describe the hormones secreted from pancreatic islets. The two important hormones secreted by islets cells of Pancreas. The hormones are: 1. Insulin 2. Glucagon. 1) Insulin Hormone: It is the active principle of the islets cells. It is albumin in nature having a molecular weight of 35,000. It is hydrolyzed by proteolytic enzymes, hence is not effective orally. It is easily destroyed by alkalies but is relatively stable in slightly acid solutions. The fact that the pancreatic tissue is rich in zinc probably indicates that insulin remains stored in islet cells as a zinc salt. The deficiency causes diabetes mellitus in man. 2) Glucagon Hormone: Kimball and Murlin (1923) demonstrated first of all the presence of glucagon hormone in the pancreatic islets. It is produced by alpha-cells of the pancreas, although a considerable amount comes from extra-hepatic alpha-cells in the stomach and other portions of the gastrointestinal tract. Glucagon is a polypeptide with a molecular weight of about 3,485. It consists of 29 amino acid having 15 different amino acids. The amino acids are arranged in a straight chain. It contains no cystine, proline or isoleucine but contains sufficient amounts of methionine and tryptophan. It can be crystallized in the absence of zinc or other metals. A pro-glucagon precursor of about 9,000 Daltons is also identified. Normal level of glucagon (fasting) in serum: 20- 100 Pg/ml. Glucagon is a diabetogenic hormone and thus is antagonistic to insulin. It helps to raising blood sugar. A glucagon-like immunoreactive factor (GLI) is also identified in gastric and duodenal mucosa but less active than pancreatic glucagon. 3) Somatostatin (SS): Somatostatin acts on the cells of the pancreas. The same substance as growth inhibiting hormone from the hypothalamus, is produced not only by the pancreas and hypothalamus but also by some cells of the digestive tract. One of the actions of somatostatin seems to suppress the release of other hormones from the pancreas. It also appears to suppress the release of hormones from the digestive tract. It may control the peristaltic activity of GI tract. It may regulate insulin and glucagon secretion by paracrine action.

Page | 73 DSC1DT: Sensory Physiology, Endocrine and Reproductive System, Renal Physiology

Compiled and circulated by Dr. Parimal Dua, Assistant Professor, Dept. of Physiology, Narajole Raj college It helps in transport of nutrients from GI tract to the circulation. It decreases HCI secretion of stomach in empty state. 4) Pancreatic Polypeptide (PP): Pancreatic polypeptide act on the cells of the pancreas. It appears that pancreatic polypeptide inhibits the release of digestive secretion of the pancreas. 42. Describe the regulation of insulin hormones secretion. Pancreatic secretion is controlled by the hormones secretin and cholecystokinin. These are produced by cells in the duodenal mucosa called amine precursor uptake and decarboxylation (APUD) cells. As the removal of pancreas spontaneously leads to diabetes mellitus, it is supposed that insulin is secreted continuously by the islets cells. However, the control of insulin (quantitatively) is affected as follows: 1) Nervous Control: There is some evidence that stimulation of vagus nerve increases insulin secretion. Normally blood sugar adjusts the vagal tone and modifies insulin secretion. High blood sugar stimulates and low blood sugar depresses the vagus nerve. But this mechanism is least effective and is only a means for fine adjustment. 2) Control by Blood Sugar Level: The blood sugar concentration of arterial blood entering the pancreas is the best controller for the secretion of insulin by islets cells. High blood sugar stimulates while low blood sugar depresses. This action is perhaps direct on the islets tissues independent of nerves. Thus, blood sugar controls insulin secretion in two ways, i.e. by direct action on the islets and through vagus nerve. However, it is also believed that the pancreotropic hormone of adenohypophysis also controls insulin secretion. But the actual way of control is still unknown. 43. Write the factors influencing secretions of glucagon. Pancreatic secretion is controlled by the hormones secretin and cholecystokinin. These are produced by cells in the duodenal mucosa called amine precursor uptake and decarboxylation (APUD) cells. 1) Low blood glucose increases secretion of pancreatic glucagon.

Page | 74 DSC1DT: Sensory Physiology, Endocrine and Reproductive System, Renal Physiology

Compiled and circulated by Dr. Parimal Dua, Assistant Professor, Dept. of Physiology, Narajole Raj college 2) Most amino acids, particularly arginine, cause a rapid secretion of glucagon from the pancreas. 3) Fatty acids inhibit glucagon release. 4) Exercise stimulates the secretion of glucagon. 5) During mixed meals, both insulin and glucagon are secreted, but the carbohydrate meal causes insulin release. High protein meal favours glucagon secretion. 6) In stress, insulin secretion is inhibited but glucagon secretion is stimulated. 44. Describe the Renin-angiotensin system and its function. Renin is a highly specific endopeptidase, whose only known function is to generate angiotensin I (Ang I) from angiotensinogen (AGT), initiating a cascade of reactions that produces an elevation of blood pressure and increases sodium retention by the kidney. Renin is an aspartyl protease that is encoded by a single gene and is first synthesized as an enzymatically inactive single polypeptide precursor molecule, prorenin. Prorenin is converted to active rennin (339–343 amino acids) after proteolytic removal of the 43 amino acid residue at the N- terminus of prorenin by several proteases. Renin is the initiative enzyme for the renin-angiotensin system. It is synthesized mainly in the juxta-glomerular apparatus of Kidney and secreted into the circulation in response to hypotension and hypernatremia. Renin is an enzyme secreted into the blood from specialized cells that encircle the arterioles at the entrance to the glomeruli of the kidneys (the renal capillary networks that are the filtration units of the kidney). The renin- secreting cells, which compose the juxtaglomerular apparatus, are sensitive to changes in blood flow and blood pressure. The primary stimulus for increased renin secretion is decreased blood flow to the kidneys, which may be caused by loss of sodium and water (as a result of diarrhea, persistent vomiting, or excessive perspiration) or by narrowing of a renal artery. Renin catalyzes the conversion of a plasma protein called angiotensinogen into a decapeptide (consisting of 10 amino acids) called angiotensin I. An enzyme in the serum called angiotensin-converting enzyme (ACE) then converts angiotensin I into an octapeptide (consisting of eight amino acids) called angiotensin II. Angiotensin II acts via receptors in the adrenal glands to stimulate the secretion of aldosterone, which stimulates salt and water reabsorption by the kidneys, and

Page | 75 DSC1DT: Sensory Physiology, Endocrine and Reproductive System, Renal Physiology

Compiled and circulated by Dr. Parimal Dua, Assistant Professor, Dept. of Physiology, Narajole Raj college the constriction of small arteries (arterioles), which causes an increase in blood pressure. Angiotensin II further constricts blood vessels through its inhibitory actions on the reuptake into nerve terminals of the hormone norepinephrine. Renin secretion by kidney is stimulated by β-adrenergic response and cAMP augmentation. It has become clear that the release of active renin from kidney is inhibited by increased arterial pressure or Ang II, and an increase in intracellular free calcium [Ca2+]. In contrast, cAMP stimulates renin release. Renin receptors bind with renin and prorenin to increase the catalytic activity of renin on angiotensinogen by 5–10 fold. Recently, renin inhibitors such as aliskiren have been developed for the treatment of hypertension. ACE inhibitors, which block the formation of angiotensin II, are used in treating high blood pressure (hypertension), which is produced by excessive constriction of the small arteries. Drugs that block the binding of angiotensin II to its receptor can also be used.

Page | 76 DSC1DT: Sensory Physiology, Endocrine and Reproductive System, Renal Physiology

Compiled and circulated by Dr. Parimal Dua, Assistant Professor, Dept. of Physiology, Narajole Raj college

Figure: Renin-angiotensin system

45. What are prostaglandins? Prostaglandins were discovered in human semen in 1935 by the Swedish physiologist Ulf von Euler, who named them, thinking that they were secreted by the prostate gland. The understanding of prostaglandins grew in the 1960s and ’70s with the pioneering research of Swedish biochemists Sune K. Bergström and Bengt Ingemar Samuelsson and British biochemist Sir John Robert Vane. The threesome shared the Nobel Prize for Physiology or Medicine in 1982 for their isolation, identification, and analysis of numerous prostaglandins. More modern terminology is use of the word "eicosanoid" instead of prostaglandin. Prostaglandins (PG) are a group of unsaturated, oxygenated fatty acids made at sites of tissue damage or infection that are involved in dealing with injury and illness. They are synthesized from a chemical called arachidonic acid via an

Page | 77 DSC1DT: Sensory Physiology, Endocrine and Reproductive System, Renal Physiology

Compiled and circulated by Dr. Parimal Dua, Assistant Professor, Dept. of Physiology, Narajole Raj college enzyme complex termed "prostaglandin synthetase" or "cyclooxygenase", which is released from the cell membrane. They are unique among hormones, because unlike most of the chemical messengers, they are not secreted from a gland. Instead, they are created at the time they are needed directly where the problem exists. Prostaglandins are not only synthesized when tissues are injured, but are ubiquitous throughout the body. They are not stored but are synthesized upon need. Once produced, prostaglandins do not persist long at their site of formation. They rapidly diffuse to other sites or are rapidly metabolized. Most prostaglandins are so rapidly metabolized; they do not reach the systemic circulation for redistribution. They control processes such as inflammation, blood flow, formation of blood clots and the induction of labour. 46. Describe the synthesis of Prostaglandins. The prostaglandins are made up of unsaturated fatty acids that contain a cyclopentane (5-carbon) ring and are derived from the 20-carbon, straight- chain, polyunsaturated fatty acid precursor arachidonic acid.

Arachidonic acid is a key component of phospholipids, which are themselves integral components of cell membranes. In response to many different stimuli, including various hormonal, chemical, or physical agents, a chain of events is set in motion that results in prostaglandin formation and release. These stimuli, either directly or indirectly, result in the activation of

an enzyme called phospholipase A2. This enzyme catalyzes the release of arachidonic acid from phospholipid molecules. Depending on the type of stimulus and the enzymes present, arachidonic acid may diverge down one of several possible pathways. One enzyme, lipoxygenase, catalyzes the conversion of arachidonic acid to one of several possible leukotrienes, which are important mediators of the inflammatory process. Another enzyme, cyclooxygenase, catalyzes the conversion of arachidonic acid to one of several possible endoperoxides. The endoperoxides undergo further modifications to form prostaglandins, prostacyclin, and thromboxanes. The thromboxanes and prostacyclin have important functions in the process of blood coagulation.

Page | 78 DSC1DT: Sensory Physiology, Endocrine and Reproductive System, Renal Physiology

Compiled and circulated by Dr. Parimal Dua, Assistant Professor, Dept. of Physiology, Narajole Raj college 47. What are the functions of prostaglandins? Biological Activities/functions of Prostaglandins: Prostaglandins have been found in almost every tissue in humans and other animals. Plants synthesize molecules similar in structure to prostaglandins, including jasmonic acid (jasmonate), which regulates processes such as plant reproduction, fruit ripening, and flowering. Prostaglandins are very potent; for example, in humans some affect blood pressure at concentrations as low as 0.1 microgram per kilogram of body weight. The structural differences between prostaglandins account for their different biological activities. Some prostaglandins act in an autocrine fashion, stimulating reactions in the same tissue in which they are synthesized, and others act in a paracrine fashion, stimulating reactions in local tissues near where they are synthesized. In addition, a given prostaglandin may have different and even opposite effects in different tissues. The ability of the same prostaglandin to stimulate a reaction in one tissue and inhibit the same reaction in another tissue is determined by the type of receptor to which the prostaglandin binds. Prostaglandins participate in a wide range of body functions such as..... 1) Vasodilation and vasoconstricton: a) A prostaglandin called prostacyclin, is acting as a vasodilator. Vasodilation occurs when the muscles in the walls of blood vessels relax so that the vessels dilate. This creates less resistance to blood flow and allows blood flow to increase and blood pressure to decrease. An important example of the vasodilatory action of prostaglandins is found in the kidneys, in which widespread vasodilation leads to an increase in the flow of blood to the kidneys and an increase in the excretion of sodium in the urine. b) Another prostaglandin called thromboxanes has the opposite effect to prostacyclins, are powerful vasoconstrictors that cause the muscle in the blood vessel wall to contract (causing the blood vessel to narrow) to decrease in blood flow and an increase in blood pressure. The opposing effects that thromboxane and prostacyclin have on the width of blood vessels can control the amount of blood flow. 2) Blood clotting: Thromboxanes and prostacyclins play an important role in the formation of blood clots. The process of clot formation begins with an aggregation of blood platelets. This process is strongly stimulated by thromboxanes and inhibited by prostacyclin.

Page | 79 DSC1DT: Sensory Physiology, Endocrine and Reproductive System, Renal Physiology

Compiled and circulated by Dr. Parimal Dua, Assistant Professor, Dept. of Physiology, Narajole Raj college a) When a blood vessel is injured, a prostaglandin called thromboxane stimulates the formation of a blood clot to try to heal the damage. Thromboxanes are synthesized within platelets, and, in response to vessel injury, which causes platelets to adhere to one another and to the walls of blood vessels thromboxanes are released to promote clot formation. Platelet adherence is increased in arteries that are affected by the process of atherosclerosis. In affected vessels the platelets aggregate into a plaque called a thrombus along the interior surface of the vessel wall. A thrombus may partially or completely block (occlude) blood flow through a vessel or may break off from the vessel wall and travel through the bloodstream, at which point it is called an embolus. When an embolus becomes lodged in another vessel where it completely occludes blood flow, it causes an embolism. Thrombi and emboli are the most common causes of heart attack (myocardial infarction). Therapy with daily low doses of aspirin (an inhibitor of cyclooxygenase) has had some success as a preventive measure for people who are at high risk of heart attack. b) In contrast, prostacyclin has the opposite effect to thromboxane, is synthesized in the walls of blood vessels and serves the physiological function of preventing needless clot formation. It reduces blood clotting and removing any clots that are no longer needed. 3) Induction of pain: Migrating cells, especially phagocytes (cells which ingest; engulf), are directly involved with pain response. During phagocytosis (the act of engulfing; ingesting), enzymes designed for the digestion of foreign material, invading agents, and damaged cells may be discharged and act against the host tissues. Release of these enzymes results in production of prostaglandins, with consequent induction of pain. High concentrations of prostaglandins cause pain by direct action upon nerve endings. More typically, however, at low concentrations, they markedly increase sensitivity to pain. The pain threshold may be so altered that even normally painless stimuli may be painful. This effect of prostaglandins is long-lasting and cumulative, so that continued production of even small amounts can sensitize nerves to other irritants. Prostaglandins are also incriminated in pain perception within the nervous system. They are produced within the central nervous system and sensitize it to painful substances. Pain is thus induced in two ways (local and central) via direct sensitization of nerve receptors by prostaglandins.

Page | 80 DSC1DT: Sensory Physiology, Endocrine and Reproductive System, Renal Physiology

Compiled and circulated by Dr. Parimal Dua, Assistant Professor, Dept. of Physiology, Narajole Raj college 4) Inflammation: Prostaglandins play a pivotal role in inflammation, a process characterized by redness (rubor), heat (calor), pain (dolor), swelling (tumor), and dysfunction of tissues and organs. The changes associated with inflammation are due to dilation of local blood vessels that permits increased blood flow to the affected area. The blood vessels also become more permeable, leading to the escape of white blood cells (leukocytes) from the blood into the inflamed tissues. Thus, drugs such as aspirin or ibuprofen that inhibit prostaglandin synthesis are effective in suppressing inflammation in patients with inflammatory but noninfectious diseases, such as rheumatoid arthritis. During an inflammatory process, infectious agents, toxins, and tissue fluids enter the circulation and cause fever. The evidence indicates this result from the production of prostaglandins in the central nervous system, specifically the anterior hypothalamus. With fever, depression (malaise) and inappetence may occur. Prostaglandins are also associated with these signs. The production of fever by prostaglandins is supported by the fact that anti- inflammatory agents which inhibit prostaglandin production may also be antipyretic. It is not, therefore, surprising that both pain and fever are the first signs of inflammation to be relieved by antiprostaglandin therapy. Swelling and redness are alleviated more slowly. 5) Role in female reproductive system: Although prostaglandins were first detected in semen, no clear role in reproduction has been established for them in males. This is not true in women, however. Prostaglandins are involved in the control of ovulation, the menstrual cycle and the induction of labour. Prostaglandins stimulate uterine muscle contraction—a discovery that led to the successful treatment of menstrual cramps (dysmenorrhea) with inhibitors of prostaglandin synthesis, such as ibuprofen. Indeed, manufactured forms of prostaglandins -

most commonly prostaglandin E2 - can be used to induce (kick-start) labour for pregnant women and they are given to induce therapeutic abortions. 6) Role in gastrointestinal tract: a) The function of the gastrointestinal tract is also affected by prostaglandins, with prostaglandins either stimulating or inhibiting contraction of the smooth muscles of the intestinal walls and may increase gut motility.

Page | 81 DSC1DT: Sensory Physiology, Endocrine and Reproductive System, Renal Physiology

Compiled and circulated by Dr. Parimal Dua, Assistant Professor, Dept. of Physiology, Narajole Raj college b) In addition, prostaglandins decrease the secretion of gastric juice, and therefore it is not surprising that drugs such as aspirin that inhibit prostaglandin synthesis may lead to peptic ulcers. c) They also increase the secretion of intestinal mucus. d) Prostaglandins increase the fluid content of the gut. They may also cause severe watery diarrhea and may mediate the effects of vasoactive intestinal polypeptide in Verner-Morrison syndrome, as well as the effects of cholera toxin. 7) Role of respiratory tract: In the lungs, prostaglandins not only modify pulmonary circulation, they also regulate the contraction and relaxation of the muscles in the airways. (Sidelight: In the newborn, as soon as the lung is functional, prostaglandins are instrumental in termination of umbilical blood flow and in the diversion of venous blood to the lung for proper aeration.) 47. How are prostaglandins controlled? The chemical reaction that makes the prostaglandins involves several steps; the first step is carried out by an enzyme called cyclooxygenase. There are two main types of this enzyme: cyclooxygenase-1 and cyclooxygenase-2. When the body is functioning normally, baseline levels of prostaglandins are produced by the action of cyclooxygenase-1. When the body is injured (or inflammation occurs in any area of the body), cyclooxygenase-2 is activated and produces extra prostaglandins, which help the body to respond to the injury. Prostaglandins carry out their actions by acting on specific receptors; at least eight different prostaglandin receptors have been discovered. The presence of these receptors in different organs throughout the body allows the different actions of each prostaglandin to be carried out, depending on which receptor they interact with. Prostaglandins are very short-lived and are broken down quickly by the body. They only carry out their actions in the immediate vicinity of where they are produced; this helps to regulate and limit their actions. 48. What happens if the levels of prostaglandins are too high? Problems with prostaglandins production can occur, leading to unwanted inflammation in the body. The prostaglandins are part of a natural response to stresses, but excessive prostaglandins production can cause chronic problems with pain. Painful menstruation, arthritis, heavy menstrual bleeding and some

Page | 82 DSC1DT: Sensory Physiology, Endocrine and Reproductive System, Renal Physiology

Compiled and circulated by Dr. Parimal Dua, Assistant Professor, Dept. of Physiology, Narajole Raj college types of cancer are all connected to excessive prostaglandins levels. Some anti- inflammatory medications work by blocking the enzymes that cause these hormones’ production, thus reducing inflammation. High levels of prostaglandins are produced in response to injury or infection and cause inflammation, which is associated with the symptoms of redness, swelling, pain and fever. This is an important part of the body’s normal healing process. However, this natural response can sometimes lead to excess and chronic production of prostaglandins, which may contribute to several diseases by causing unwanted inflammation. This means that drugs, which specifically block cyclooxygenase-2, can be used to treat conditions such as arthritis, heavy menstrual bleeding and painful menstrual cramps. There is also evidence to suggest that these drugs may have a beneficial effect when treating certain types of cancer, including colon and breast cancer, however research in this area is still ongoing. New discoveries are being made about cyclooxygenases which suggest that cyclooxygenase-2 is not just responsible for disease but has other functions. Anti-inflammatory drugs, such as aspirin and ibuprofen, work by blocking the action of the cyclooxygenase enzymes and so reduce prostaglandin levels. This is how these drugs work to relieve the symptoms of inflammation. Aspirin also blocks the production of thromboxane and so can be used to prevent unwanted blood clotting in patients with heart disease. 49. What happens if the levels of prostaglandins are too low? Sometimes, the body will not create enough prostaglandins to heal the injury or start labor. While this is not connected to any chronic health condition, artificial prostaglandins can help. In fact, manufactured prostaglandins can be used to increase prostaglandin levels in the body under certain circumstances. For example, administration of prostaglandins can induce labour at the end of pregnancy or abortion in the case of an unwanted pregnancy. They can also be used to treat stomach ulcers, glaucoma and congenital-heart-disease'>congenital heart disease in newborn babies. 50. What is erythropoietin? Erythropoietin, also known as erythropoetin, haematopoietin, or haemopoietin, is a 30-kDa glycoprotein hormone secreted mainly by the kidney of adults in response to cellular hypoxia (low oxygen levels in the blood). Erythropoietin has 165 amino acids. Erythropoietin stimulates red blood cell production in the

Page | 83 DSC1DT: Sensory Physiology, Endocrine and Reproductive System, Renal Physiology

Compiled and circulated by Dr. Parimal Dua, Assistant Professor, Dept. of Physiology, Narajole Raj college bone marrow. The erythropoietin hormone level can be detected and measured in the blood (the EPO test). Normal levels of erythropoietin range from 4 up to 24 mU/ml (milliunits per milliliter). Low levels of erythropoietin (around 10 mU/mL) are constantly secreted sufficient to compensate for normal red blood cell turnover. Common causes of cellular hypoxia resulting in elevated levels of erythropoietin (up to 10,000 mU/mL) include any anemia, and hypoxemia due to chronic lung disease. Erythropoietin is produced by interstitial fibroblasts in the kidney in close association with the peritubular capillary and proximal convoluted tubule. It is also produced in perisinusoidal cells in the liver. Liver production predominates in the fetal and perinatal period; renal production predominates in adulthood. It is homologous with thrombopoietin. 51. What is the role of erythropoietin? Erythropoietin (EPO) is a hormone produced primarily by the kidneys. Erythropoietin is produced to a lesser extent by the liver. Only about 10% of erythropoietin is produced in the liver. It plays a key role in the production of red blood cells (RBCs), which carry oxygen from the lungs to the rest of the body. The EPO test measures the amount of erythropoietin in the blood. Erythropoietin is produced and released into the blood by the kidneys in response to low blood oxygen levels (hypoxemia). Erythropoietin is carried to the bone marrow, where it stimulates production of red blood cells. The hormone is active for a short period of time and then eliminated from the body in the urine. The amount of erythropoietin released depends upon how low the oxygen level is and the ability of the kidneys to produce erythropoietin. Increased production and release of erythropoietin continues to occur until oxygen levels in the blood rise to normal or near normal concentrations, then production falls. The body uses this dynamic feedback system to help maintain sufficient oxygen levels and a relatively stable number of RBCs in the blood. However, if a person's kidneys are damaged and do not produce sufficient erythropoietin, then too few RBCs are produced and the person typically becomes anemic. Similarly, if a person's bone marrow is unable to respond to the stimulation from erythropoietin, then the person may become anemic. This can occur with some bone marrow disorders or with chronic diseases, such as rheumatoid arthritis.

Page | 84 DSC1DT: Sensory Physiology, Endocrine and Reproductive System, Renal Physiology

Compiled and circulated by Dr. Parimal Dua, Assistant Professor, Dept. of Physiology, Narajole Raj college Individuals who have conditions that affect the amount of oxygen they breathe in, such as lung diseases, may produce more erythropoietin to try to compensate for the low oxygen level. People who live at high altitudes may also have higher levels of erythropoietin and so do chronic tobacco smokers. If too much erythropoietin is produced, as occurs with some benign or malignant kidney tumors and with a variety of other cancers, too many RBCs may be produced (polycythemia or erythrocytosis). This can lead to an increase in the blood's thickness (viscosity) and sometimes to high blood pressure (hypertension), blood clots (thrombosis), heart attack, or stroke. Rarely, polycythemia is caused by a bone marrow disorder called polycythemia vera, not by increased erythropoietin. 52. What is melatonin ? Melatonin is a neurohormone found naturally in the body. Alternative name of melatonin is N-acetyl-5-methoxytryptamine. Melatonin is produced by the pineal glands in the brain, mainly at night. It prepares the body for sleep and is sometimes called the “hormone of sleep” or “hormone of darkness.” The production and release of melatonin from the pineal gland occurs with a clear daily (circadian) rhythm, with peak levels occurring at night. Once produced, it is secreted into the blood stream and cerebrospinal fluid (the fluid around the brain & spinal cord) and conveys signals to distant organs. Melatonin is carried by the circulation from the brain to all areas of the body. Tissues expressing proteins called receptors specific for melatonin are able to detect the peak in circulating melatonin at night and this signals to the body that it is night-time. Night-time levels of melatonin are at least 10-fold higher than daytime concentrations. Sleep isn’t the only body function affected by melatonin. This hormone also plays a role in the body’s antioxidant defenses and helps regulate blood pressure, body temperature and cortisol levels, as well as sexual and immune function. 53. How does melatonin work? Melatonin's main job in the body is to regulate night and day cycles or sleep-wake cycles. Darkness causes the body to produce more melatonin, which signals the body to prepare for sleep. Light decreases melatonin production and signals the body to prepare for being awake. Some people who have trouble sleeping have low levels of melatonin. It is thought that adding melatonin from supplements might help them sleep.

Page | 85 DSC1DT: Sensory Physiology, Endocrine and Reproductive System, Renal Physiology

Compiled and circulated by Dr. Parimal Dua, Assistant Professor, Dept. of Physiology, Narajole Raj college In addition to its circadian rhythm, melatonin levels also have a seasonal (or circannual) rhythm, with higher levels in the autumn and winter, when nights are longer, and lower levels in the spring and summer. In many animals (including a wide range of mammals and birds), melatonin from the pineal gland is essential for the regulation of the body’s seasonal biology (e.g. reproduction, behaviour and coat growth) in response to changing day length. The importance of pineal melatonin in human biology is not clear, although it may help to synchronise circadian rhythms in different parts of the body. In humans, nocturnal levels of melatonin decrease across puberty. The level of circulating melatonin can be detected in samples of blood and saliva, and this is used in clinical research to identify internal circadian rhythms. Most of the research into the function of the pineal gland involves the human brain's responses to melatonin rhythms. The evidence supports two roles for melatonin in humans: the involvement of nocturnal melatonin secretion in initiating and maintaining sleep, and control by the day/night melatonin rhythm of the timing of other 24-hour rhythms. Melatonin has, therefore, often been referred to as a ‘sleep hormone’; although it is not essential for human sleep, we sleep better during the time that melatonin is secreted. Association between tumours of the pineal gland and the timing of puberty suggests that melatonin may also have a minor role in reproductive development, although the mechanism of this action is uncertain. Melatonin secretion by the human pineal gland varies markedly with age. Melatonin secretion starts during the third or fourth months of life and coincides with the consolidation of night-time sleep. Following a rapid increase in secretion, nocturnal melatonin levels peak at ages one to three years, then decline slightly to a plateau that persists throughout early adulthood. After a steady decline in most people, night-time levels of melatonin in a 70-year old are only a quarter or less of those seen in young adults. Night-time melatonin secretion is suppressed by a relatively dim light when pupils are dilated. This has been suggested as the main way through which prolonged use of devices such as laptops and smartphones before bedtime can have a negative impact on melatonin secretion, circadian rhythms and sleep. Melatonin used as medicine is usually made synthetically in a laboratory. In addition to its production in the body, melatonin can also be taken in capsule form, but melatonin is also available in forms that can be placed in the cheek or

Page | 86 DSC1DT: Sensory Physiology, Endocrine and Reproductive System, Renal Physiology

Compiled and circulated by Dr. Parimal Dua, Assistant Professor, Dept. of Physiology, Narajole Raj college under the tongue. This allows the melatonin to be absorbed directly into the body. Some people take melatonin by mouth to adjust the body's internal clock. Melatonin helps you fall asleep, improve sleep quality and increase sleep duration. Melatonin is most commonly used for insomnia and improving sleep in different conditions. For example, it is used for jet lag, for adjusting sleep-wake cycles in people whose daily work schedule changes (shift-work disorder), and for helping people establish a day and night cycle. When administered at an appropriate time of day, it can reset the body’s circadian rhythms. This resetting effect of melatonin has been reported for many dose strengths, including those that are equivalent to the concentration of melatonin naturally produced by the pineal gland. Higher doses of melatonin can reset circadian rhythms, bring on sleepiness and lower core body temperature. 54. How is melatonin controlled? Melatonin is a natural hormone made by your body's pineal gland. This is a pea- sized gland located just above the middle of the brain. During the day the pineal is inactive. In humans and other mammals, the daily rhythm of pineal melatonin production is driven by the 'master' circadian clock. This 'clock' is in a region of the brain called the suprachiasmatic nuclei, which expresses a series of genes termed clock genes that continuously oscillate throughout the day. This is synchronised to the solar day via light input from the eyes. The suprachiasmatic nuclei link to the pineal gland through a complex pathway in the nervous system, passing through different brain areas, into the spinal cord and then finally reaching the pineal gland. Besides adjusting the timing of the clock, bright light has another effect. It directly inhibits the release of melatonin. During the day, the suprachiasmatic nucleus stops melatonin production by sending inhibitory messages to the pineal gland. That is why melatonin is sometimes called the "Dracula of hormones" - it only comes out in the dark. Even if the pineal gland is switched "on" by the clock, it will not produce melatonin unless the person is in a dimly lit environment. In addition to sunlight, artificial indoor lighting can be bright enough to prevent the release of melatonin. When the sun goes down and darkness occurs, the pineal is "turned on" by the suprachiasmatic nucleus (SCN) and begins to actively produce melatonin, which is released into the blood. At night however, the suprachiasmatic nuclei are less active, and the inhibition exerted during the day is reduced resulting in melatonin production by the pineal gland. Usually, this occurs around 9 pm. As

Page | 87 DSC1DT: Sensory Physiology, Endocrine and Reproductive System, Renal Physiology

Compiled and circulated by Dr. Parimal Dua, Assistant Professor, Dept. of Physiology, Narajole Raj college a result, melatonin levels in the blood rise sharply and you begin to feel less alert. Sleep becomes more inviting. Melatonin levels in the blood stay elevated for about 12 hours - all through the night - before the light of a new day when they fall back to low daytime levels by about 9 am. Daytime levels of melatonin are barely detectable. Light is an important regulator of melatonin production from the pineal gland. Firstly, it can reset a specific area of the brain (the suprachiasmatic nuclei clock) and, as a result, the timing of the melatonin production. Secondly, exposure to light during the body's biological night reduces melatonin production and release. 55. How human sleep is regulated in exposure to light or to darkness ? Exposure to light stimulates a nerve pathway from the retina in the eye to an area in the brain called the hypothalamus. There, a special center called the suprachiasmatic nucleus (SCN) initiates signals to other parts of the brain that control hormones, body temperature and other functions that play a role in making us feel sleepy or wide awake. The SCN works like a clock that sets off a regulated pattern of activities that affect the entire body. Once exposed to the first light each day, the clock in the SCN begins performing functions like raising body temperature and releasing stimulating hormones like cortisol. The SCN also delays the release of other hormones like melatonin, which is associated with sleep onset, until many hours later when darkness arrives. 56. What happens if you have too much melatonin? There are large variations in the amount of melatonin produced by individuals and these are not associated with any health problems. The main consequences of swallowing large amounts of melatonin are drowsiness and reduced core body temperature. Very large doses have effects on the performance of the human reproductive system. There is also evidence that very high concentrations of melatonin have an antioxidant effect, although the purpose of this has not yet been established. 57. How to increase melatonin levels naturally ? You can increase your melatonin levels without supplementing.  A few hours before bedtime, simply dim all lights at home and avoid watching TV and using your computer or smartphone.  Too much artificial light can reduce the production of melatonin in the brain, making it harder for you to fall asleep (23Trusted Source).

Page | 88 DSC1DT: Sensory Physiology, Endocrine and Reproductive System, Renal Physiology

Compiled and circulated by Dr. Parimal Dua, Assistant Professor, Dept. of Physiology, Narajole Raj college  You can also strengthen your sleep-wake cycle by exposing yourself to plenty of natural light during the day, especially in the morning (24Trusted Source).  Other factors that have been associated with lower natural melatonin levels include stress and shift work. 58. Discuss about different cells and their secretary hormones of gastrointestinal tract. Gastrointestinal hormones are peptide hormones secreted by endocrine cells, which are widely distributed throughout the mucosa of the gastrointestinal tract. These hormones regulate intestinal and pancreatic functions, by affecting secretion, motility, absorption, digestion, and cell proliferation. Therefore, these hormones could be challenging targets to affect the rate of digestion and absorption of starch. A Endocrine/Paracrine Cells Cells containing gastrointestinal hormones are found scattered along the gut in all regions of the epithelium, although they show a preferential localization toward the base of the mucosa. The cells share the features of possessing secretory granules and lying against the basal lamina. The morphology of the cells falls into three general types: (1) flask shaped, with the apex reaching the lumen and covered in microvilli, (2) similar to (1) but with elongations progressing along the basal lamina, and (3) lacking lumenal contact, being covered by nonendocrine cells.

Page | 89 DSC1DT: Sensory Physiology, Endocrine and Reproductive System, Renal Physiology

Compiled and circulated by Dr. Parimal Dua, Assistant Professor, Dept. of Physiology, Narajole Raj college 1) Enterochromaffin (EC) Cells: One of the most abundant endocrine cell types in the gastrointestinal tract is in the enterochromaffin (EC) cell, which can be found, in differing numbers, in all regions of the gut (Figure 1). The cells are usually flask-shaped, but they sometimes have basal elongations, particularly in the gastric fundus. The EC cells are argentaffin and argyrophil, and, immunocytochemically, they are characterized by their content of serotonin (5-hydroxytryptamine or 5-HT). They have been reported also, in various species, to contain peptides, including substance P, motilin , and enkephalin. The secretory granules in EC cells are characteristically electron dense and very osmiophilic, with a non-spherical shape which has been reported to be biconcave.

Figure 1: Enterochromaffin cells revealed in human colonic mucosa by immunostaining of serotonin (5- hydroxytryptamine), using the peroxidase–anti-peroxidase (PAP) technique. Bar, 10 μm.

2) Enterochromaffin-like (ECL) Cells: Enterochromaffin-like (ECL) cells, in contrast to EC cells, are confined to a single region of the gut, the gastric fundus, where they form the major endocrine cell population. ECL cells have no lumenal connection (Figure 2). The cells are argyrophil but nonargentaffin. They were initially recognized as early as 1945 and were later characterized by Solcia and co-workers. ECL cells can be revealed by Sevier and Munger's silver impregnation technique or by immunostaining of general endocrine cell markers, such as chromogranin. No peptide product has been identified as yet in human ECL cells, although those of rodents are known to secrete histamine, which can be demonstrated using specific antibodies or antibodies against the enzyme histidine decarboxylase. Under the electron microscope, ECL cells are

Page | 90 DSC1DT: Sensory Physiology, Endocrine and Reproductive System, Renal Physiology

Compiled and circulated by Dr. Parimal Dua, Assistant Professor, Dept. of Physiology, Narajole Raj college easily distinguishable from EC cells. They usually contain numerous, large vesicles within which a core of variable electron density is surrounded by a thin halo and membrane.

Figure 2: The results of Sevier and Munger's silver impregnation technique showing the typical appearance of an ECL cell, lying along the basal lamina and not connecting with the lumen (L). Bar, 10 μm. Nomarski optics.

3) Gastrin (G) and Intestinal Gastrin (IG) Cells: In the stomach, gastrin (G) and somatostatin (D) cells are found, along with glucagon cells in some species such as dog. Gastrin is confined to the antrum (Figure 3) and upper small intestine. G cells were first described in 1967 and were characterized by their ultrastructural features of variably sized, electron-lucent secretory granules. A minor population of smaller, more electron-dense granules can also be seen. Gastrin occurs in more than one form. Pyloric G cells predominately contain the G17 form, whereas the larger G34 form of gastrin is found in the intestinal gastrin (IG) cells of the upper small intestine. The IG cells, in contrast to their gastric counterpart, contain mainly small, electron- dense granules.

Page | 91 DSC1DT: Sensory Physiology, Endocrine and Reproductive System, Renal Physiology

Compiled and circulated by Dr. Parimal Dua, Assistant Professor, Dept. of Physiology, Narajole Raj college

Figure 3: Gastrin (G) cells in human antral mucosa immunostained by the PAP method. Bar, 20 μm. Nomarski optics.

Page | 92 DSC1DT: Sensory Physiology, Endocrine and Reproductive System, Renal Physiology

Compiled and circulated by Dr. Parimal Dua, Assistant Professor, Dept. of Physiology, Narajole Raj college 4) Somatostatin (D) Cells: In 1975, somatostatin was localized to gut cells which had been previously termed D cells on the basis of their ultrastructural similarity to the D cells of the pancreas. These cells have large, round, and uniform secretory granules. The D cells can be found in all regions of the gut but are present in the largest numbers in the stomach, and colon. Somatostatin is, perhaps, the most typical paracrine hormone. In accordance with its functional role, the peptide can be found in cells with basal elongations, along which secertory granules are transported to release their content to neighboring cells. Somatostatin can also be found in the gut innervation of some species. 5) Motilin (M), Secretin (S), GIP (K), Cholecystokinin (I), and Neurotensin (N) Cells: The variety of endocrine cells becomes greater in the duodenum and jejunum. In addition to the EC, G, and D cells, those containing motilin, secretin, gastric inhibitory peptide (GIP), and cholecystokinin (CCK) are found (Figure 4).

Figure 4: Low power micrograph of human duodenal mucosa showing scattered CCK-immunoreactive cells. Bar, 50 μm.

Page | 93 DSC1DT: Sensory Physiology, Endocrine and Reproductive System, Renal Physiology

Compiled and circulated by Dr. Parimal Dua, Assistant Professor, Dept. of Physiology, Narajole Raj college There appear to be two major molecular forms of motilin in both blood and tissue. Both types appear to be stored in typical, flask-shaped cells of the duodenum (Figure 5) and jejunum, but some EC cells in addition contain just the larger of the two forms, which can be detected immunocytochemically using antibodies to the amino terminus of motilin. Under the electron microscope, the motilin (M) cell is characterized by small, round granules with a closely applied membrane. This ultrastructural profile is somewhat similar to that of the IG cells, but serial sections show that the intestinal gastrin and motilin cells are completely separate cell types.

Figure 5: Motilin immunoreactivity in an endocrine cell adjacent to a goblet cell in the human duodenal mucosa. Bar, 20 μm. Nomarski optics.

Secretin and GIP are structurally related to each other and to glucagon and VIP. The two peptides are found in the duodenum and, to a lesser extent, the jejunum, where they are produced by the typically endocrine S and K cells, respectively. The S cell contains relatively small types of secretory granules. The granules are rather irregular in shape and have a characteristic small halo between the electron-dense core and its surrounding membrane. The GlP- containing K cells, in contrast, have large, mostly round but sometimes irregular, dense granules. Although cholecystokinin, in its smaller molecular form, has been reported in the enteric nerves, it is present primarily in mucosal endocrine cells of the duodenum and jejunum. The peptide, which can be found in several molecular forms, is secreted by the I cell which contains intermediate-sized (hence the term I cell), electron-dense, round granules.

Page | 94 DSC1DT: Sensory Physiology, Endocrine and Reproductive System, Renal Physiology

Compiled and circulated by Dr. Parimal Dua, Assistant Professor, Dept. of Physiology, Narajole Raj college The ileum is the major location of the N cell, the source of neurotensin . The N cell has a typical endocrine, flasklike shape and contains large, round granules. 6) Enteroglucagon (EG) Cells: The most numerous cell type in the ileum, colon, and rectum is the so-called EG (enteroglucagon) or glicentin-containing cell (Figure 6). The ultrastructural profile of the EG cell appears to correspond to that previously described for the L cell , with its large, round, and electron- dense granules with a heavily argyrophilic rim. Under the light microscope, these cells can often be seen with long, basal elongations, similar to those described for somatostatin-containing cells.

Figure 6: Human colonic mucosa containing an enteroglucagon-immunoreactive cell with a luminal connection. Bar, 20 μm. Nomarski optics.

The product of the EG cells was thought originally to be solely glicentin, but the precursor form of glicentin, preproglucagon, has been isolated and characterized. Along with glicentin, the molecule also gives rise to pancreatic glucagon, the glucagon-related peptides and, at the carboxy terminus, the glucagonlike peptides 1 and 2. Immunocytochemistry has shown, at both the light and electron microscopical levels, that the EG cells contain GLP- 1 and GLP-2, as well as glicentin. In fact, serial sections through a single cell show that each secretory granule contains all three peptides. A further peptide has also been localized to the EG cell. Peptide tyrosine tyrosine (PYY) is a member of the pancreatic polypeptide (PP) family of structurally related peptides. It has a limited distribution in the ileum and colon, where is it found in EG cells.

Page | 95 DSC1DT: Sensory Physiology, Endocrine and Reproductive System, Renal Physiology

Compiled and circulated by Dr. Parimal Dua, Assistant Professor, Dept. of Physiology, Narajole Raj college 59. Discuss about gastrointestinal hormones. Gastrointestinal hormones, commonly referred to as gut hormones, are a group of hormones secreted by specific cells called enteroendocrine cells or endocrinocytes that are scattered throughout the digestive system. Also, these enteroendocrine cells possess hormone-containing granules concentrated at the basolateral membrane, adjacent to capillaries, which secrete their hormones via exocytosis in response to a wide range of stimuli related to food intake. These stimuli include small peptides, amino acids, fatty acids, oral glucose, distension of an organ, and vagal stimulation. All of the gastrointestinal hormones are peptides, which are chains of amino acids and very similar to proteins. The hormones control a number of different functions of the digestive organs. The GI hormones classify as endocrines, paracrine, or neurocrine based on the method by which the molecule gets delivered to its target cell(s).  Endocrine hormones are secreted from enteroendocrine cells directly into the bloodstream, passing from the portal circulation to the systemic circulation, before being delivered to target cells with receptor-specificity for the hormone. There are six gastrointestinal hormones that are generally recognized as the endocrines. They include hormones from the secretin, Gastrin-cholecystokinin, and motilin families. Hormones are grouped together based on their chemical structure, and often perform similar functions. A. Gastrin-cholecystokinin family: The gastrin-cholecystokinin family includes gastrin and cholecystokinin. 1) Gastrin: G cells secrete gastrin in the antrum of the stomach and the duodenum in response to the presence of breakdown products of protein digestion (such as amino acids and small peptides), distention by food, and vagal nerve stimulation via GRP. More specifically, phenylalanine and tryptophan are the most potent stimulators of gastrin secretion among the protein digestion products. The vagal nerve stimulation of gastrin secretion is unique because gastrin and motilin are the only hormones released directly by neural stimulation. Gastrin stimulates the growth of the epithelium in the gastric tube, and also stimulates gastric acid secretion. 2) Cholecystokinin (CCK): Cholecystokinin (CCK) is secreted from enteroendocrine I cells in the duodenum and jejunum in response to acids and monoglycerides (but not triglycerides), as well as the presence of protein digestion products. The physiological roles of CCK are the stimulation of pancreatic secretion and the contraction of the gall-bladder. Cholecystokinin

Page | 96 DSC1DT: Sensory Physiology, Endocrine and Reproductive System, Renal Physiology

Compiled and circulated by Dr. Parimal Dua, Assistant Professor, Dept. of Physiology, Narajole Raj college stimulates the gallbladder to contract and empty in response. It also stimulates the pancreas to secrete enzymes that break down the peptides and fatty acids in the small intestine. In addition, CCK is abundantly expressed in the brain and functions as a neurotransmitter. There are two types of CCK receptors: CCK-Ar and CCK-Br, both of which are G protein–coupled receptors derived from different genes. These receptors are expressed in various central and peripheral tissues including brain, stomach, pancreas, kidney, and vagal afferent fibers, although expression patterns and function of these receptors may be different among tissues and species. Activation of bitter taste receptors in the intestine leads to secretion of CCK, which is proposed to help limit the absorption of dietary-derived bitter-tasting toxins. Released CCK from the intestine may activate CCK receptors on the vagus nerve, leading to suppression of food intake and gastric emptying. B. Secretin family: Secretin and gastric inhibitory peptide (GIP) are part of the secretin family, and is produced as a result of acidic pH in the small intestine. 1) Secretin: Secretin is secreted from enteroendocrine S cells in the duodenum in response to H+ and fatty acids in the lumen. Specifically, a pH less than 4.5 signals arrival of gastric contents, which initiates the release of secretin. Secretin stimulates secretion of water and bicarbonate in the pancreas and the bile ducts. 2) Gastric inhibitory peptide (GIP): GIP is secreted by enteroendocrine K cells in the duodenum and jejunum in response to glucose, amino acids, and fatty acids. GIP is the only GI hormone with a response to all three macronutrient types, and newer studies suggest that changes in intraluminal osmolarity may be what stimulates GIP secretion. GIP responds to elevated blood glucose levels in the small intestine, and it inhibits the motility of the small intestine. It also stimulates the beta cells in the pancreas to release insulin. C. Motilin family: Ghrelin and motilin are part of the motilin family. It is unclear what stimulates them to be secreted, but they are associated with certain physiological states. 1) Ghrelin: Ghrelin seems to be a stimulant of appetite and feeding, as its secretion peaks right before feeding and drops once there is gastric filling. It also strongly stimulates growth hormone secretion. Ghrelin is a gastrointestinal hormone that stimulates feeding and secretion of growth

Page | 97 DSC1DT: Sensory Physiology, Endocrine and Reproductive System, Renal Physiology

Compiled and circulated by Dr. Parimal Dua, Assistant Professor, Dept. of Physiology, Narajole Raj college hormone (GH). Ghrelin is thought to directly affect neurons involved in feeding or GH secretion through growth hormone secretagogue receptor (GHS-R; ghrelin receptor); however, it is still unclear whether ghrelin crosses through the blood–brain barrier. Recently, several gastrointestinal hormones have been shown to transmit signals involved in feeding to the brain at least in part via the vagal afferent system. In fact, ghrelin's action on feeding or GH secretion is abolished or attenuated in rats that have undergone vagotomy or treatment with capsaicin, a specific afferent neurotoxin. GHS-R is present in the vagal afferent neurons as well as the brain and is transported to the afferent terminals. In addition, the firing rate of vagal afferent fibers significantly decreases after ghrelin administration. Taken together, these data show that the vagal afferent system is the major pathway conveying ghrelin's signals for feeding and GH secretion to the brain. 2) Motilin: Motilin seems to be associated with fasting, and it also seems to help keep motility of the stomach and small intestine. It directly stimulates motility complexes within the DT, particularly the duodenum and jejunum. Indirectly, it stimulates the release of acetylcholine. Reflexes occurring at different points along the DT can alter the contractile activity in the various parts of the small intestine. We mention four: (1) gastroileal reflex, (2) gastric evacuation, (3) increases of segmentation in the ileum, and (4) intestinointestinal reflex. This last reflex can completely shut down motility in cases of significant distension of the large intestine, including from constipation, lesions of the intestinal wall, or presence of bacterial infections.  Enteroendocrine cells also secrete paracrine hormones, but they diffuse through the extracellular space to act locally on target tissues and do not enter the systemic circulation. Two examples of paracrine hormones are somatostatin and histamine. 1) Somatostatin: Somatostatin was localized to gut cells which had been previously termed D cells on the basis of their ultrastructural similarity to the D cells of the pancreas. These cells have large, round, and uniform secretory granules. The D cells can be found in all regions of the gut but are present in the largest numbers in the stomach, and colon. Somatostatin is, perhaps, the most typical paracrine hormone. In accordance with its functional role, the peptide can be found in cells with basal elongations, along which secertory

Page | 98 DSC1DT: Sensory Physiology, Endocrine and Reproductive System, Renal Physiology

Compiled and circulated by Dr. Parimal Dua, Assistant Professor, Dept. of Physiology, Narajole Raj college granules are transported to release their content to neighboring cells. Somatostatin can also be found in the gut innervation of some species. 2) Histamine: Enterochromaffin-like (ECL) cells are confined to a single region of the gut, the gastric fundus. No peptide product has been identified as yet in human ECL cells, although those of rodents are known to secrete histamine, which can be demonstrated using specific antibodies or antibodies against the enzyme histidine decarboxylase.  Additionally, some hormones may operate via a combination of endocrine and paracrine mechanisms. These “candidate” hormones are glucagon-like peptide-1 (GLP-1), pancreatic polypeptide, and peptide YY. 1) Glucagon-like peptide-1 (GLP-1): GLP-1 is secreted from enteroendocrine L cells mainly located in the distal small intestine and colon. The presence of hexose and fat stimulate its release. It stimulates the insulin secretion, decreases the secretion of glucagon and delays gastric emptying. 2) Pancreatic polypeptide: Pancreatic polypeptide is secreted by protein and fat. 3) Peptide tyrosine-tyrosine (PYY): Peptide YY are secreted by protein and fat. The biological actions of peptide tyrosine-tyrosine (PYY) include inhibition of pancreatic secretion and contraction of the gallbladder. In addition, PYY has been postulated as an agent involved in the ileal brake phenomenon and in the regulation of food intake.  Lastly, neurocrine hormones get secreted by postganglionic non-cholinergic neurons of the enteric nervous system. Three neurocrine hormones with significant physiologic functions in the gut are vasoactive intestinal peptide (VIP), gastrin release peptide (GRP), and enkephalins. 61. Describe the physiological functions of gastrointestinal hormones. The two gastrointestinal hormone families discussed above are responsible for most of the regulation of gastrointestinal function. The main actions of the gastrin-CCK family and the secretin family of hormones are listed below. 1) Gastrin i. Stimulates H+ (acid) secretion by parietal cells in the stomach ii. Trophic (growth) effects on the mucosa of the small intestine, colon, and stomach iii. Inhibits the actions of Secretin and GIP iv. Inhibited by H+ 2) CCK

Page | 99 DSC1DT: Sensory Physiology, Endocrine and Reproductive System, Renal Physiology

Compiled and circulated by Dr. Parimal Dua, Assistant Professor, Dept. of Physiology, Narajole Raj college i. Contraction of the gallbladder with simultaneous relaxation of the sphincter of Oddi ii. Inhibits gastric emptying iii. Stimulates secretion of pancreatic enzymes: lipases, amylase, and proteases iv. Secretion of bicarbonate from the pancreas v. Trophic effects on the exocrine pancreas and gallbladder 3) Secretin i. It inhibits gastrin and H+ secretion, and growth of stomach mucosa ii. It helps promote the release of bicarbonate from the bile ducts. Bicarbonate is something that neutralizes the stomach acid that was just flushed into the duodenum from the stomach. Think of bicarbonate as pouring water on the fire that is stomach acid. iii. It stimulates the pancreas to start secreting bicarbonate juice iv. Trophic effect on the exocrine pancreas 4) GIP i. Stimulation of insulin secretion ii. Induces satiety iii. In large doses, decreases gastric acid secretion iv. In large doses, decreases the motor activity of the stomach and therefore slows gastric emptying when the upper small intestine is already full of food products. v. Stimulates the activity of lipoprotein lipase in adipocytes vi. Protects beta-cells of the pancreas from destruction by apoptosis 5) GLP-1 i. Decreases gastric emptying ii. Induces satiety iii. Increases sensitivity of pancreatic beta-cells to glucose. 6) Motilin i. Increases gastrointestinal motility by stimulating the “migrating motility” or “myoelectric complex” that moves through the fasting stomach and small intestines every 90 minutes. This cyclical release and action get inhibited by the ingestion of food. Not much is known about this peptide, except for this essential function.

Page | 100 DSC1DT: Sensory Physiology, Endocrine and Reproductive System, Renal Physiology

Compiled and circulated by Dr. Parimal Dua, Assistant Professor, Dept. of Physiology, Narajole Raj college 62. Discuss the release mechanisms of gastrointestinal hormones. Most gastrointestinal hormones are released after a meal to allow or facilitate the digestion and absorption of nutrients. Motilin is a unique hormone; it is released periodically during the interdigestive fasting period, and its cyclical release is abolished after a meal. Therefore, a “biological clock,” still of unknown nature, somewhere in the organism periodically signals motilin cells to release the peptide into the circulation. In vitro preparations of intestinal mucosal cells enriched in motilin cells showed that muscarinic receptors are present on the motilin cell membrane and that protein kinase C activators are the most potent second messengers eliciting motilin release. In the ex vivo perfused canine intestine, bombesin has been identified as a direct stimulant of motilin release, whereas the stimulatory effect of opiates was mediated by acetylcholine. However, phenylephrine and somatostatin seem to act directly on M cell membrane receptors to block the release of the peptide. Interactions with luminal serotonin13 as well as plasma ghrelin25 on the release of plasma motilin have been suspected recently. Vagus nerves seem to have some influence on the release of plasma motilin because acute vagal cooling inhibited the cyclical peak increases in plasma motilin; however, vagotomized dogs displayed a normal cyclical profile of motilin plasma release.

Figure: Schematic representation of canine plasma motilin variations. During the fasting interdigestive period, motilin is released cyclically every 80–120 min (lower panel) to induce the phase III contraction of the migrating motor complex (MMC) from the stomach to the ileum (indicated by the dark boxes in the upper panel). After eating, motilin cyclical peak increases are abolished for 2–8 h (depending on the content and nature of the meal) while the MMC is interrupted and the fed pattern motility profile is taking place.

Page | 101 DSC1DT: Sensory Physiology, Endocrine and Reproductive System, Renal Physiology

Compiled and circulated by Dr. Parimal Dua, Assistant Professor, Dept. of Physiology, Narajole Raj college

As the stimulation of motilin cyclical peak release during the interdigestive period is incompletely understood, inhibition of this cyclical release by a meal (Fig. 2) is also incompletely understood. The situation is still more complicated by the fact that meal ingestion is followed in humans by a very early and brief increase in plasma motilin before the interdigestive release cycle is interrupted, as in the dog. This early release in humans can be mimicked experimentally by central stimulation with modified sham feeding and by distension of the gastric fundus with an air-filled balloon. The contribution of this postprandial motilin release, not present in the dog, in the process of nutrient digestion in humans remains to be characterized. 63. What is Atrial natriuretic peptide or atrial natriuretic factor (ANP or ANF) ? Write its functions. Atrial natriuretic peptide (ANP) is a hormone that is synthesized from cells (atrial myocytes) in the right atrium of the heart and is released when right atrial pressure increases. The release of this cardiac peptide is stimulated by increases in the stretch of the atrial wall caused by an increase in blood pressure or blood volume. ANF receptors are also stimulated by elevated sodium levels. ANP has an intra-molecular ring structure connected by two cysteine residues and N- terminal and C-terminal extensions from it. ANP is present in mammals, amphibians and bony fishes, but absent in birds, reptiles, cartilaginous fishes, and cyclostomes. Types of atrial natriuretic factor: Atrial natriuretic factor (ANF) or atrial natriuretic peptide (ANP) consists of mixtures of peptides of various lengths. Studies have shown that the circulating form of ANF in plasma consists of a 28-amino acid residue ANF (ANP), whereas the rat and human atrium largely stores pro-ANF (γ-ANP), containing 126 amino acid residues, formed from prepro-ANF. The pro-ANF converts to the 28-amino acid active peptide by a co-secretional mechanism. Analysis by high-performance liquid chromatography (HPLC) revealed that ANF is stored in the brain tissues in active forms consisting of 24 and 25 residues. ANF in urine increases with increased salt feeding, reflecting its well-recognized role in natriuresis. However, it has been found that urinary ANF contains 32 amino acid residues. Purification studies revealed that two additional forms of ANF exist, which are products of different genes. Therefore, three different genes have been found for ANF families, and they are termed ANP (ANF), BNP, and

Page | 102 DSC1DT: Sensory Physiology, Endocrine and Reproductive System, Renal Physiology

Compiled and circulated by Dr. Parimal Dua, Assistant Professor, Dept. of Physiology, Narajole Raj college CNP. BNP was originally isolated from porcine brain but was subsequently shown to exist largely in the atria and ventricles of the heart, whereas CNP seems to be localized in the brain. Atrial natriuretic factor (ANF) and B-type (BNP) and C-type (CNP) natriuretic peptides (NPs) are homologous peptides that share common receptors and exert similar actions in the periphery, all related to the control of fluid and electrolyte homeostasis and cardiovascular function. Functions of Atrial natriuretic factor: 1) ANP induces profound natriuresis/diuresis, hypotension, and inhibition of aldosterone secretion in mammals. 2) ANF induces profound natriuresis and diuresis by the kidneys. It promotes salt and water excretion. It acts on receptors that increase GFR, decrease NaCl reabsorption in the distal nephron, and inhibit renin secretion, resulting in overall water loss. ANP is released during volume expansion and contributes to the natriuretic response. 3) Atrial natriuretic peptide (ANP) and B-type natriuretic peptide (BNP) act at many sites in the kidney and elsewhere to decrease left atrial pressure and blood pressure and increase renal salt excretion. 4) ANP lowers blood pressure. 5) The actions of ANF that tend to reduce cardiac output. 6) ANF also causes vasodilatation and reduces fluid volume by acting directly on vascular smooth muscle and inhibiting the release norepinephrine from peripheral adrenergic neurons, that result lowers blood pressure. 7) ANF inhibits angiotensin II and vasopressin, resulting in vasodilatation and decrease in blood volume and blood pressure. 8) ANF extremely potent hormone enhances salt and water excretions by inhibiting the release of aldosterone from the adrenal cortex. 9) Additionally, ANF has been found in the brain, and centrally medicated effects on fluid volume regulation may be important. 10) In brain, ANF inhibits vasopressin secretion, sympathetic tone, thirst, and salt appetite.

Page | 103 DSC1DT: Sensory Physiology, Endocrine and Reproductive System, Renal Physiology